Download as pdf or txt
Download as pdf or txt
You are on page 1of 60

14

Pain management

197
CARE OF THE CRITICALLY ILL SURGICAL PATIENT

including how it is generated and perceived.


OBJECTIVES As a member of the surgical team, you will look
This chapter will help you to: after patients presenting with primarily painful
conditions and will encounter pain produced
• understand the effects and risks of
by surgical operations and interventions.
inadequate analgesia
• learn to function as a member of a Safe and effective management of acute pain is
multidisciplinary team providing analgesia an integral part of surgical practice. Although
in the critically ill patient the physical status of the patient, the degree of
• understand your role in the management trauma and the available techniques may be
of acute pain throughout the continuum of very different, the principles involved are similar
surgical care from the general ward through whether the patient is recovering from major
to HDU/ICU surgery on a general ward or being managed
• acquire the knowledge to decide on on an HDU or ICU.
the appropriate method of analgesia and Good quality analgesia is essential for humanitarian
the skills necessary for your role in its reasons alone: there are also compelling medical
deployment reasons for its provision. Inadequately controlled
• be aware of the systemic effects of pain increases sympathetic outflow leading to
analgesia and the complications of analgesic an increase in heart rate, vasoconstriction and
techniques. increased oxygen demand, particularly in the
myocardium where it may contribute to ischaemia.
It may impair lung function; abdominal and
thoracic procedures almost always lead to impaired
INTRODUCTION respiratory function because the pain induced by
Nociception is the name given to the reception movement inhibits coughing and diaphragmatic
of a noxious or unpleasant stimulus and its function leading to atelectasis/pneumonia. The
conversion into an impulse transmitted through patient who has adequate analgesia is able to
a sensory nerve. Pain refers to the perception of mobilise and this will reduce the incidence of
that sensation in the CNS and its recognition as deep vein thrombosis, improve respiratory function
unpleasant. Suffering is the emotional response and improve general psychological well being.
that makes one miserable and is associated with It should also be recognised that pain
the severity of the pain and its duration. postoperatively may be caused by a developing
Society relies on doctors to provide relief of surgical complication.
pain and suffering. It is important for all doctors However, a degree of pain is not always bad.
to have a fundamental understanding of pain Pain causes people to rest and protect injured
tissue, preventing further damage and allowing
healing.

198
CHAPTER 14 | PAIN MANAGEMENT

PRINCIPLES OF required. Any patient who has escalating analgesic


ACUTE PAIN MANAGEMENT requirements needs to be assessed with a high
The realistic aim of pain relief is not to abolish degree of suspicion (see Case Scenario 14.1).
pain in the postoperative period totally but to Ischaemia is a common trap, as are bleeding,
ensure that patients are comfortable and have anastomotic leakage and compartment syndrome.
return of function with a more rapid recovery ‘Breakthrough pain’ in a patient who previously
and rehabilitation. There are several important had effective analgesia should be treated as a
principles relevant to the provision of good surgical complication until proven otherwise.
quality pain relief.
MANAGE EXPECTATION
PREVENT The two most powerful forces in play during pain
The single most important step we can take management are the expectation of the patient
in alleviating pain is to prevent the factors and that of the healthcare staff. If either believe
that produce it. Avoiding tension during surgical that the patient will be in severe pain, the outcome
closure may help, as may preventing drains will tend towards poorer mobilisation and poorer
or tubes from pulling on sutures or relieving outcomes; if both believe the converse, the outcome
urinary retention. is likely to be fewer complaints and more rapid
mobilisation.
The use of drugs preventing the development of
pain is more effective than treatment of existing Pre-operatively, patients should be told to expect
established pain. This is the concept of pre-emptive significant pain in proportion to their condition.
analgesia. In practical terms, this means that local They should also be told that pain can be
anaesthetic drugs or other analgesic agents should controlled by various means and that they will not
be given prior to surgical trauma rather than be allowed to suffer. If distressed by their pain, or
afterwards. if pain is inhibiting their breathing or their ability
to cough, a change in technique will be required!
RECOGNISE NEW PROBLEMS
It is critical that trainee surgeons are able to SURGICAL CONSIDERATIONS
recognise when a patient’s pain has altered to Upper abdominal incisions are associated
a point where an alternative explanation is with considerably more pain, more pulmonary

CASE SCENARIO 14.1


A 75-year-old man has worsening abdominal pain 3 days after low anterior resection. He is requiring
continued high-dose PCA morphine and is now nauseated and vomiting with a tender abdomen.
An anastomotic breakdown and collection in the abdomen needs to be excluded as the patient should
be having decreasing analgesic requirements by this time. The nausea and vomiting may be due to
obstruction not opiate. This patient needs a full clinical assessment to deduce the cause of deterioration
and increased analgesia requirement.

199
CARE OF THE CRITICALLY ILL SURGICAL PATIENT

disturbance and more difficulty effecting adequate


analgesia than lower or transverse abdominal PRACTICE POINT
incisions. The choice of site and type of incision As the clinician likely to be contacted first
is therefore important in the sick patient who has in the case of a surgical patient becoming
limited respiratory reserve. critically ill, the surgical trainee must
Laparoscopic surgery is considerably less traumatic establish if:
and so less painful during the postoperative course. (i) pain is related to a surgical complication;
Postoperative pain management will be aided (ii) poor pain relief is contributing to the
by the use of local anaesthetic infiltration at the patient’s lack of progress; and
time of surgery or by specific local and regional (iii) the method of analgesia is contributing to
anaesthetic techniques. Epidural infusion analgesia the patients deterioration.
is the most commonly used of these techniques;
other examples include caudal blocks for perineal
surgery, intercostal blocks for cholecystectomy, PATIENT ASSESSMENT
and ilio-inguinal blocks for lower abdominal AND MANAGEMENT
incisions. Upper and lower limb surgery can have
IMMEDIATE MANAGEMENT
major nerve blocks or plexus blocks to provide
In the critically ill patient in pain, patient
postoperative analgesia.
assessment is vital. It should follow the same
CCrISP system of assessment as in any other
THE ROLE OF THE SURGICAL circumstance.
TRAINEE IN THE MULTIDISCIPLINARY
ACUTE PAIN TEAM Airway
The provision of postoperative pain relief has Start at the beginning by checking that the
always been hindered by confusion about whose patient has a patent airway. Over sedation
responsibility it is to perform this function. secondary to opioid drugs may be associated
Traditionally, postoperative analgesia has been with episodic airway obstruction. This is
prescribed by the anaesthetist, administered by the particularly marked in patients who are elderly,
ward nurses and supervised by a junior surgical obese, have a history of obstructive sleep apnoea
trainee. The Joint College Working Party Report (OSA) or who have had surgery to the head or
on Postoperative Pain recommended that each neck. It may be exacerbated by the administration
major hospital should have a multidisciplinary acute of other sedative drugs such as benzodiazepines.
pain team consisting of surgeons, anaesthetists,
nursing staff and pharmacists. With increasingly
sophisticated methods of analgesia being used, it
is vitally important that the surgical trainee liaises
with the other members of the team and is aware
of protocols and guidelines relating to acute pain
management in the hospital.

200
CHAPTER 14 | PAIN MANAGEMENT

In the assessment of the critically ill patient


PRACTICE POINT in pain, it is therefore essential to assess the
Episodes of airway obstruction and resultant adequacy and depth of the respiratory pattern as
hypoxaemia often persist for 2–3 nights after well as respiratory rate, and to check the patient’s
major surgery. Supplemental oxygen should ability to cough. Investigations including ABG
be continued for at least 72 hours in high-risk analysis and continuous pulse-oximetry can
patients recovering from major surgery who be useful adjuncts to clinical findings when
receive any form of opioid analgesia (including assessing a patient’s respiratory adequacy
PCA and by the epidural route). (see Case Scenario 14.2).

Circulation
Breathing Tachycardia should not automatically be assumed
Check the respiratory rate, pattern and depth of to be caused by pain – there is commonly an
breathing. Is your patient’s respiratory function underlying cause.
impaired by inadequate analgesia? Can he or she A persistent tachycardia or hypertension caused
cough and expectorate properly to avoid problems by inadequate analgesia may potentiate the
later? The rational use of opioid analgesia has development of myocardial ischaemia, particularly
always been limited by the fear of drug-induced in the patient who is already hypoxaemic.
respiratory depression. A much more common A common clinical problem is the differential
problem is the patient slowly slipping into respiratory diagnosis of hypotension occurring in the
failure due to poorly controlled pain which is post-surgical patient. This may be due to any
inhibiting movement and the ability to cough. cause of shock, from simple hypovolaemia due
to inadequate fluid input or bleeding, through
cardiac failure due to CCF, ischaemia and
PRACTICE POINT infarction or arrhythmias and septic shock.
Respiratory rate is a late and unreliable The patient may also be receiving epidural
indicator of opiate-induced respiratory analgesia and this may cause added confusion
depression. Sedation levels are a more sensitive as to the cause of hypotension, due to the relative
indicator of impending opioid overdosage. hypovolaemia secondary to vasodilatation.
Severe hypoxaemia may occur in the presence Patients with sympathetic blockade are very
of normal or usually raised respiratory rates. sensitive to inadequate volume replacement and
Poorly relieved pain, particularly in upper care must be taken in these patients to replace
abdominal surgery is a major cause of failure fluid losses immediately.
to cough, sputum retention and hypoxaemia. This requires meticulous attention to the
Remember pulse oximetry oxygen saturations maintenance of accurate fluid balance charts,
are not a good guide to respiratory function, measurements of losses from surgical drains and
only to oxygenation. a high index of suspicion for concealed losses.

201
CARE OF THE CRITICALLY ILL SURGICAL PATIENT

FULL PATIENT ASSESSMENT


PRACTICE POINT Chart review
Persistent hypotension and tachycardia in If pain relief is felt to be contributing to the
the post-surgical patient may be due to any patient’s deterioration, the drug charts should be
cause of shock. All types of shock need to reviewed with the following questions in mind:
be considered, investigated and excluded. • is effective analgesia prescribed?
Epidural analgesia should not be assumed • is effective analgesia being given?
automatically to be the cause of hypotension. • is the treatment appropriate for this patient?
The recorded pain and sedation scores should be
reviewed and the scores repeated (see below).
Disability
It is important to assess whether the method of History and systemic examination
analgesia is contributing to the patient’s clinical The contribution of pain to the patient’s general
deterioration. Particular attention should be paid condition should be ascertained during your full
to the patient’s level of consciousness as patient assessment.
decreasing conscious level is an early indicator
of opioid toxicity. Behavioural observations
Assessment of pain by hospital staff is usually
based on the patient’s outward response, or ‘pain
behaviour’. Verbal complaints, facial expression,
restriction of mobility and changes in heart rate
and blood pressure are used intuitively to build an
impression of how much pain a patient is suffering.
While these are often good predictors of pain, it is
important to realise that, in some individuals,

TABLE 14.1

PAIN SCORING SYSTEMS

Verbal rating scale Is your pain: 0, absent; 1, mild; 2, discomforting; 3, distressing;


4, excruciating
Numerical rating scale Which number describes your pain: 0, no pain; 10, worst imaginable
Visual analogue scale No pain to Worst imaginable
Functional assessment Can you move? Can you cough?

202
CHAPTER 14 | PAIN MANAGEMENT

such assessments may be quite wrong and we Decide and plan


may significantly under- or over-estimate the If pain relief is adequate and the patient is
level of suffering. improving then continue and review. If pain
relief is inadequate determine why:
Severity scoring systems • is it due to failure of the method of analgesia?
The effectiveness of assessment of analgesia will • is it due to incorrect implementation of the
be vastly increased if simple reproducible pain method chosen?
scoring systems are used. These systems emphasise • is it due to the development of a surgical
restoration of function by assessing pain scores complication?
during movement and when coughing. (Table Pain relief should be considered as an integral
14.1.) Pain scores should be recorded when the part of the patient’s total care. In certain situations,
patient is taking deep breaths, coughing and on the method of analgesia may determine whether
movement otherwise the score may be falsely low. the patient has a smooth postoperative course
or becomes critically ill. In a patient who has
PRACTICE POINT pre-existing chest disease, for example, the analgesic
technique chosen when an upper abdominal
Pain scoring in the patient recovering from
incision is planned may determine whether a
major surgery should be as performed routinely
period of postoperative ventilation on an ICU is
along with measurement of cardiovascular
required or not.
and respiratory parameters.
Increasing pain scores should alert you to
potential complications. TECHNIQUES AVAILABLE FOR
THE MANAGEMENT OF ACUTE PAIN
It is important for all those involved in the delivery
Investigations
of pain services to understand the range of
Investigations assessing respiratory function are
techniques available. At one end of this spectrum
frequently used when assessing the adequacy of
is the administration of single analgesic agents,
analgesia. Serial ABG analysis and chest X-rays
often administered orally, to a patient in mild
are often used to demonstrate trends in respiratory
discomfort. As the intensity of pain increases,
capacity and sputum cultures are essential when
there is a need for an increased response from
planning antibiotic therapy. Remember that pulse
those providing the pain relief. In some cases,
oximetry only tells you about oxygenation not
increased analgesic requirements may be met by
overall respiratory function – the reading of
increasing the dose or potency of the drugs used.
saturation should be interpreted in the context of
Other situations will demand the use of more
inspired oxygen concentration and respiratory rate.
sophisticated regimens. Combinations of methods
and agents may be needed (multimodal therapy).

203
CARE OF THE CRITICALLY ILL SURGICAL PATIENT

Alternatively, effective analgesia may require the


use of sophisticated techniques, such as patient
controlled analgesia (PCA) or epidural infusion
analgesia (EIA). In all surgical procedures there
may be appropriate local anaesthetic nerve/plexus
blocks or simple wound infiltration with local Epidural
anaesthetic drugs used to reduce pain. PCA analgesia
It may be helpful to think of the increasing level
of intervention required with increasing pain in
terms of the WHO analgesic ladder. As a patient’s
pain intensity escalates, so does the level of Multimodal
support needed (Fig. 14.1, Table 14.2). When therapy
the situation improves and the intensity of the Single-agent
pain decreases, analgesic requirements will also analgesia
decrease and a technique from lower down the
ladder can be used.

Figure 14.1 Analgesic ladder.

TABLE 14.2

PAIN INTENSITY ESCALATION AND MANAGEMENT

Pain intensity Management


Mild Paracetamol or NSAID
Mild-to-moderate Combination analgesic ± NSAID
Moderate Oral opioid OR combination analgesic ± NSAID

Moderate-to-severe Oral opioid + paracetamol ± NSAID


Severe Parenteral opioid (i.v., i.m. or s.c.) + paracetamol ± NSAID OR epidural
(local anaesthetic ± opioid)

Combination analgesics are a mixture of weak oral opioid and paracetamol (see next page).

204
CHAPTER 14 | PAIN MANAGEMENT

ANALGESIC AGENTS
PRACTICE POINT Paracetamol
The use of a technique from higher up This is a very useful drug that has a high
the analgesic ladder does not necessarily therapeutic index and very few side effects in
mean stopping more simple methods; normal dosage. It is toxic in overdose because it
epidural analgesia can often be effectively depletes the glutathione reserve of the liver and
supplemented by the use of regular then damages hepatocytes. Paracetamol should
paracetamol or NSAIDs. be given regularly and can be administered by
oral, rectal or, more recently, the intravenous
route (as the pro-drug pro-paracetamol). It should
Opioids should only be administered by one route form the basis of most in-hospital pain regimens.
at a time – respiratory and other toxic effects
from epidural opioids will be potentiated if oral, Non-steroidal anti-inflammatory drugs (NSAIDs)
intramuscular or intravenous opioids are given NSAIDs are increasingly used as part of balanced
concurrently. Such toxicity is potentially fatal! analgesia as adjuncts to opioid analgesia in an
attempt to increase efficacy and reduce opioid
SINGLE AGENT ANALGESIA side effects. Different preparations are available
Except in minor pain or discomfort, it is unusual for dosing by sublingual, oral, rectal and parenteral
for optimal analgesia to be obtained from a single routes. This group of drugs is unlikely to be
agent or technique. If single agent analgesia is chosen for the management of pain relief in the
used, it will be more effective if drugs are critically ill patient due to effects on haemostasis
prescribed and administered regularly rather than and renal function.
on an ‘as-required’ basis. All analgesic drugs can
be given as single agents but are usually more
PRACTICE POINT
effective when given as part of a balanced
multimodal therapy regimen. NSAIDs are often contra-indicated in critically
ill patients due to their potentially disastrous
MULTIMODAL THERAPY effects on renal function and gastric mucosa.
It is often difficult to produce safe, effective NSAIDs are absolutely contra-indicated in the
analgesia with a single group of drugs. Better elderly hypotensive patient.
results with fewer side effects are achieved if
combinations of drugs affecting different parts
of the pain pathway are used. Such ‘balanced The COX-2 inhibitors are a newer subgroup;
analgesia’ (multimodal therapy) usually consists however, they are substantially more expensive
of a combination of local anaesthetics, opioids than standard NSAIDs and accumulating evidence
and NSAIDs, and paracetamol. suggests that gastrointestinal side effects may
not be substantially different and efficacy is no
greater. They should not be used in patients with
ischaemic heart disease.

205
CARE OF THE CRITICALLY ILL SURGICAL PATIENT

Opioids Once analgesia is achieved, i.v. dosing is


Codeine phosphate is an opioid with weak analgesic commonly maintained by self-administration of
properties and, in randomised controlled trials, further doses using patient controlled analgesia
has an efficacy equal to paracetamol in adequate (see below).
dose. However, it is profoundly constipating and
may produce substantial nausea which limits its Intravenous opioid infusions
usefulness. It is often used in combination Although opioid infusions can be very effective,
preparations with paracetamol. respiratory and other side effects are common,
Tramadol is a stronger opioid than codeine. It has and they should only be used in an intensive care
many opioid-like properties but without the same setting. Intravenous opioid infusions are beyond
extent of respiratory depression or the tendency the scope of the CCrISP course and will not be
to produce dependence. It does however have a considered further.
marked emetic effect in many patients.
Intramuscular opioids
Although the bio-availability of morphine is quite
The intermittent ‘as-required’ prescription of
low by the oral route, adequate effect can easily
intramuscular opioid analgesia was the traditional
be achieved with dose titration. It carries with it
form of postoperative pain relief. However, the
all the usual potential side effects of opioids.
use of 4-hourly intramuscular injections has
Patients with short-term needs or in transition to repeatedly been shown to be ineffective in a high
oral medication are best managed with morphine proportion of patients recovering from abdominal
elixir; for chronic pain, a slow release tablet form surgery and this technique has now been
may be most appropriate. Other opioids are also superceded by more effective methods of analgesia.
available orally or as slow release transcutaneous It is also painful for the patient and has unreliable
patches (fentanyl). absorption in the hypovolaemic patient.
Opioids remain the gold standard of analgesia However a single one-off i.m. dose may allow you
when a potent agent is required for severe pain. to relieve pain relatively quickly and safely while
Morphine is the cheapest and most widely available organising a more definitive analgesic regimen.
agent. The principles discussed apply to all opioids,
as do the toxic effects. Opioid side effects
The limiting factor for use in the conscious post-
Intravenous opioids – bolus doses operative patient is the emergence of side effects
Adequate analgesia is most rapidly achieved by in a dose-dependent progression. All opioids
an intravenous bolus dose, or repeated doses if the exhibit similar effects, although the profiles of
desired effect is not achieved with the initial dose. different agents may differ in the detail.
Often, boluses of 5 mg are given initially, followed
by further increments of 1–2 mg until satisfactory Respiratory depression
analgesia is achieved. This may require surprisingly All opioids reduce the sensitivity of the respiratory
large doses; 20–30 mg is commonly needed in an centre in the brainstem in a dose-dependent manner.
average sized patient to produce good pain relief. Even in therapeutic doses, the partial pressure of

206
CHAPTER 14 | PAIN MANAGEMENT

carbon dioxide (PaCO2) will show an elevation Remember that the half-life for an intravenous
from the normal value (5.3 kPa = 40 mmHg). dose of naloxone is short (approximately 15–20
A high normal (PaCO2) of up to 6.5 kPa should minutes) and symptoms may re-appear.
be considered an expected consequence of using
such drugs (as should constricted pupils) and is Nausea and vomiting
not a reason to stop using them. Both respiratory This is a distressing and common side effect.
rate and tidal volume are affected by opioids. It is dose-related and is potentiated by movement
Respiratory rate is easier to measure at the and when gastric emptying is already impaired.
bedside and it is extremely unlikely that dangerous It is caused both by direct stimulation of the
respiratory depression will occur without a fall chemoreceptor trigger zone (CTZ) in the medulla
in rate below the normal range (12 breaths/min and by gastric distention. Common anti-emetics
in adults). available are ondansetron, cyclizine, metoclopramide
and prochlorperazine. Dexamethasone may also
Sedation help to relieve nausea.
Decreasing level of consciousness carries with it
the risk of loss of protective reflexes, especially PRACTICE POINT
those associated with protection of the airway
Nausea and vomiting may be due to a
(cough, gag and the ability to recognise imminent
surgical cause rather than analgesic regimen
regurgitation). As unconsciousness deepens,
or anaesthesia.
airway occlusion may occur.

PRACTICE POINT PATIENT CONTROLLED ANALGESIA


A decreased level of consciousness is a The technique is based on the concept of the
medical emergency if the GCS is less than 8 patient self-administering a bolus dose of
due to imminent loss of protective reflexes morphine intravenously (usually 1 mg), after
and ability to maintain the airway. which the PCA machine will allow no further
demands for a predetermined period – the
patient ‘lock-out’ time. During this lock-out
A low respiratory rate will usually improve
period, which is usually set at 5 minutes, the
by stopping administration of opioids. If the
patient is unable to receive further doses. After
respiratory rate falls below 8 breaths/min, if
the ‘lock-out’ time has elapsed, the patient is
the patient becomes hypoxic or is at risk from
able to repeat the dose of analgesic drug if they
a decreased level of consciousness, small doses
are still in pain. In this way, the patient titrates
of the opioid antagonist naloxone may be
their own level of analgesia, increasing demands
indicated (100 mcg repeated until the desired
when requirements are high (during physiotherapy
effect is achieved). Be aware that naloxone may
for example) and reducing them when needs
produce dysphoria, hypertension, tachycardia
are lower or if they experience side effects.
and the sudden and unwelcome return of severe
pain if not carefully titrated.

207
CARE OF THE CRITICALLY ILL SURGICAL PATIENT

CASE SCENARIO 14.2


You review a 52-year-old man on the morning of the second postoperative day following a repair
of an incisional hernia which had occurred in an old upper midline scar. Initial assessment shows
the patient to be well built (95 kg), a little drowsy but adequately rousable. His breathing is rapid
(24 breaths/min) and shallow and he cannot breathe deeply enough for you to hear his breath
sounds well. He is sweaty, tachycardic (110 bpm) but normotensive and well perfused. A pulse
oximeter showed SaO2 of 88% initially – this has risen to 92% with mask oxygen at 6 l/min. He
has a past history of smoking and mild chronic bronchitis. Review of his charts shows that 4-hourly
morphine 10 mg has been given for analgesia.
You think that he is hypoxic, largely because of poor analgesia.
You increase the flow rate of oxygen to 12 l/min and ask for it to be humidified. The SaO2 increases
to 98%. The patient’s conscious level improves and he affirms that he is in pain from his wound.
It is 90 min since his last analgesia so you give a further 10 mg i.m. morphine and 20 min later he
is more comfortable – auscultation now reveals reasonable air entry but some expiratory wheeze.
Salbutamol (2.5 mg in 5 ml) is given by nebuliser.
Blood gases (on 12 l/min oxygen) show: PaO2 20 kPa (FiO2, 0.6); PaCO2 7 kPa; pH 7.29;
BE 0.4mmol/l; HCO3– 28mmol/l.
There is a mild respiratory acidosis and hypercarbia and an acceptable oxygen concentration.
Note that the SaO2 has not told you about the raised PaCO2.
You arrange for review by the on-call physiotherapist and for a chest radiograph. After discussion
with the pain team, arrangements are made for PCA to be established and you prescribe regular
paracetamol 6-hourly either p.o or i.v, and p.o. or p.r. diclofenac 50 mg 8-hourly (having previously
noted that he has normal renal function).
You review the patient at lunchtime. Repeat blood gases 1 h after physiotherapy show marked
improvement. With continuous pulse oximetry, the oxygen flow is steadily reduced to 6 l/min.
The patient is comfortable, breathing well and able to cough at will.

LEARNING POINTS
• poor analgesia is common and has profound effects on respiratory and other vital organ
function – 4-hourly prn opiates are often inadequate
• analgesic techniques are generally better at preventing pain than at rescuing a patient from
marked discomfort with associated complications
• review the effect of interventions – re-assess your patient!
• a multidisciplinary approach can be very useful in pain management.

208
CHAPTER 14 | PAIN MANAGEMENT

PCA is well accepted by patients and nursing PCA is unsuitable for patients who are confused
staff, gaining high levels of patient satisfaction or who are unable to press the demand button for
and providing good quality analgesia. The efficacy physical reasons.
and safety of the technique depends upon the
factors shown in Table 14.3. EPIDURAL ANALGESIA (EA)
The most effective way of producing profound
TABLE 14.3 analgesia is to block afferent pain pathways by
the use of epidurally administered local anaesthetic
CHECK LIST FOR PCA drugs.

Does the patient understand PCA? Yes


Insertion of the epidural catheter
Do the staff understand PCA? Yes This procedure is commonly performed by the
anaesthetist at the time of surgery. Both lumbar
Is the patient the only person pressing Yes
and thoracic approaches are used, the latter being
the button?
commonly used to provide analgesia for both
Is the pain responsive to opioids? Yes thoracic and abdominal surgery.

Is the patient receiving any other No


PRACTICE POINT
opioid or sedative drug?
Many anaesthetists will not insert an epidural
Is the patient receiving a background No catheter if the patient has received a dose
infusion? of anticoagulant prophylaxis within the last
Is the patient being monitored Yes 12 hours. Consult with the anaesthetist if
appropriately? pre-operative heparin or LMWH are being
considered.

The major advantage of PCA is that it gives the


patient control of their analgesia and greatly ADMINISTRATION OF EPIDURAL DRUGS
reduces the fear of unrelieved pain. It is also The anaesthetist will have checked the position
intrinsically safe; if the patient becomes sedated, of the epidural catheter (ensured that it is in the
they will administer no further drug, blood levels epidural space and excluded accidental subdural
will fall, and they will recover consciousness. or subarachnoid placement) by giving a test dose
Sleep disturbance is a major problem with use of local anaesthetic and will have established an
of PCA. The requirement for regular dosing may infusion before the patient leaves the theatre suite.
prevent the patient having adequate periods of The infusion will usually be a mixture of a dilute
undisturbed sleep. Often, the pain is severe by the concentration of local anaesthetic (bupivacaine
time the patient re-awakens and takes some time 0.1–0.125%) with small amounts of added opioid
to bring under control. (commonly fentanyl 2–4 mcg/ml). Combinations of
drugs are administered for similar reasons to other
routes – better analgesia with fewer side effects.

209
CARE OF THE CRITICALLY ILL SURGICAL PATIENT

CASE SCENARIO 14.3


8 hours after an uncomplicated right total knee replacement, you are asked to review a 59-year-old
woman with ischaemic heart disease who has gradually become hypotensive (BP 80/40) and
tachycardic 110 sinus rhythm.
You assess the patient and find her alert, comfortable and with acceptable perfusion. Continuous
epidural analgesia is in progress, using a mixture of fentanyl and bupivacaine in standard dosage.
There has been no chest pain, dysrhythmia or hypoxia (lowest SaO2 95%). Total volume in the
drains is 750 ml, which the specialist nurse relates as ‘average’. The wound is not soaked nor the leg
swollen and the patient has received saline at 150 ml/h since returning from theatre. The epidural
appears to be working well – the block is infra-umbilical, the patient can move her toes and is
totally pain free. The infusion is running at 12 ml/h.
You conclude that the patient is hypovolaemic but cannot decide whether postoperative blood loss
or vasodilatation from the epidural is predominant. The patient says she feels thirsty (a good sign
of hypovolaemia). In either event, a fluid challenge is needed and you give 500 ml colloid over 15
min with little effect. During this time, a full blood count shows Hb 11.0 g/dl. You stop the epidural
infusion.
Peripheral perfusion improves but blood pressure fails to respond. There are no signs of on going
bleeding or narcotisation (which can occasionally occur). An ECG shows no acute change. Given
the history of ischaemic heart disease, you are reluctant to give a further 500 ml fluid challenge
so opt for 250 ml over 15 min. The BP begins to rise to 90/50 and the pulse drops to 100. You opt
to give another fluid bolus of 250 ml colloid over 15 min and see the BP rise to 110/60 and pulse
drop to 90. The epidural block is now decreasing and you decide to re-start the infusion at 6 ml/h
and re-assess in 15 min continuing the maintenance fluids.
An hour later, the BP is maintained, there is good urine output indicating adequate renal perfusion
and the patient is comfortable. You make plans to review the patient in 2 h.

LEARNING POINTS
• hypotension associated with epidural analgesia is common – sympathetic blockade,
peri-operative bleeding and loss of fluid into tissues or through insensible losses can all
contribute – and investigations are needed to establish the cause
• postoperative bleeding must be actively considered and dealt with
• the need for treatment of hypotension due to the epidural alone depends on the level of
the blood pressure, co-morbidity (especially cardiac or peripheral vascular) and the effect
on end organs (urine output)
• all patients should be adequately filled with intravenous fluid – monitoring by CVP on
HDU may be required in the patient with multiple co-morbidities.

210
CHAPTER 14 | PAIN MANAGEMENT

The aim is to establish good pain relief with other causes, especially hypovolaemia, bleeding,
minimal sympathetic effects and no motor block. myocardial infarction and sepsis (see Case
Infusion rates of 8–15 ml/h are commonly used. Scenario 14.3).
A functioning epidural gives outstanding pain Once it has been established that epidural-induced
control. vasodilatation is the cause of the hypotension,
the infusion should be reduced and adequate fluid
Troubleshooting epidurals resuscitation should be undertaken to correct the
The two most common problems are breakthrough relative hypovolaemic state.
pain and hypotension. Both may also be due to
evolving surgical complications. Close co-operation OTHER LOCAL/REGIONAL TECHNIQUES
between the surgical team and the acute pain There are numerous other techniques based on
service along with clear management protocols the use of local anaesthetic agents, which may be
are essential. encountered in specific circumstances. These range
from simple intra-operative infiltration of the
Breakthrough pain wound to nerve blocks, plexus blocks, intrapleural
This may be due to a problem with the epidural infusions and so on. All have their proponents
or the development of a new surgical problem. and specific indications, but all work on the
The patient should be fully assessed on each principle of blocking generation or conduction
occasion. Help should be sought from the pain of the noxious stimulus to prevent its being
team if it is apparent that the epidural is not perceived as pain.
functioning. An increase in the infusion rate
(often preceded by a bolus or ‘top-up’ dose) may
be required or the catheter may require PRACTICE POINT
re-positioning. Patients may return to the ward with local
anaesthetic infusions – these are severely toxic
Hypotension if infused intravenously.
Hypotension is a relatively common problem This also applies to epidural local anaesthetic
with epidural infusions, particularly in younger infusions.
patients and in those with higher level blocks.
If hypotension is caused by the epidural, it is
usually due to sympathetic block and consequent CHRONIC PAIN
vasodilatation. As there are many other common This is beyond the scope of this text, except to
causes of hypotension in the postoperative note that poor management of acute surgical pain
patient, the epidural must not be assumed to may be one factor in the production of chronic
be responsible until other potential causes have pain and chronic analgesic dependence.
been excluded. As always, when assessing the Paradoxically, the latter is more often produced
hypotensive patient with an epidural, use the by inadequate use of analgesics rather than over
CCrISP system of assessment to avoid missing use, as is commonly believed.

211
CARE OF THE CRITICALLY ILL SURGICAL PATIENT

WHERE SHOULD SUCH PATIENTS SUMMARY


BE NURSED? • you are an essential member of the acute
pain team, often being the first person called
For many reasons, including the provision of
to a pain related problem
adequate postoperative pain relief, all patients
• when assessing the patient in pain, use the
recovering from major surgery should be nursed
CCrISP system of assessment
in an area with a high ratio of nursing staff to
• poor pain relief threatens the critically ill patient
patients. Any ward designated for the care of
• pain may be due to evolving surgical
patients recovering from major surgery should
complications
have enough trained nurses and doctors to care
• pain relief should provide comfort and
for patients requiring PCA or epidural analgesia
restoration of function; analgesia should be
subject to the following provisions:
assessed using reproducible pain scores and
the ability of the patient to cough effectively
1. The establishment of an acute pain service • multimodal therapy can dramatically improve
with named consultants responsible for the pain relief and reduce side effects: remember
provision of postoperative pain relief. the concept of the analgesic ladder
• PCA and epidural analgesia are very effective
2. Rapid 24-hour availability of designated
techniques but patients need careful monitoring
doctors and resuscitation team.
and regular re-assessment to prevent problems
3. A system for monitoring patients on a regular • be in regular contact with the acute pain team
basis including pain scores, respiratory rate or your anaesthetic colleagues!
and sedation scores.
4. Protocols and an education programme for
all staff for the detection and management
of major complications.
5. Availability of continuous monitoring
or transfer to an HDU or ICU for high-risk
cases.

Provision of adequate analgesia is often difficult


and complex in the sick surgical patient:
consideration should be given to transferring
such patients to a higher level of care.

212
15
Communication,
organisation
and leadership
in surgical care

213
CARE OF THE CRITICALLY ILL SURGICAL PATIENT

patient may be. It is easy to make assumptions of


OBJECTIVES what other people think and believe, which often
reflects one’s own beliefs rather than the reality
This chapter will help you to:
for that individual. Many problems arising in
• understand the importance of clear surgical care are the result of poor communication,
and effective communication in surgical often serial, rather than a lack of knowledge or
critical care an incorrect decision. There is evidence to suggest
• appreciate some of the barriers to that adverse outcomes, iatrogenic injuries, failure
communication and ways of overcoming these to provide adequate care, mistakes, providing
• develop an increased awareness of the incorrect care or system errors are more likely
leadership role, which the surgical registrar to lead to litigation or complaints if there have
plays in the surgical team been preceding communication problems. Other
• appreciate the value of personal and team international data on litigation have shown
organisation in facilitating this adverse outcomes occurring in 3.7% of admissions
• understand how people normally deal with 1 in 4 (1% of total) due to negligence.
with adverse events and appreciate the However, two out of three claims come from
frequency with which serious traumatic patients with no adverse outcome or an adverse
events can lead to stress reactions. outcome not due to negligence. Another study
has shown that only 3% of patients who suffered
negligence filed a lawsuit. Reasons given for
instigating litigation include a desire to correct
It is a self-evident fact that being an effective apparent deficient standards of care, to find out
communicator is a vital skill for all surgeons. what happened and why, to enforce accountability
The way care is provided and the expectations and to gain compensation for accrued and future
of patients and relatives have changed markedly costs. A further study has shown that 70% of
in recent times, posing new and challenging litigation is related to poor communication, citing
issues for surgical teams. Large numbers of people, feelings such as desertion, devaluation, lack of
coming from different aspects of the healthcare information and lack of understanding. In one
professions are now involved in the care of a study, over half of patients who commenced
single patient and this process needs to be actively litigation claimed that they were so unimpressed
managed by the surgical consultant and his or by the doctor that they wanted to sue him/her
her team, who have ultimate and continuing before the alleged event occurred.
responsibility for each individual patient. Patients For the surgical team to function at its best, and
and relatives, quite rightly, expect good outcomes thereby stand any chance of delivering optimum
from surgical interventions and expect to be kept levels of care consistently, there must be good
informed about details of their care at each stage. communication, organisation and leadership within
It is vitally important to understand patients’ the immediate team and with others. To achieve
expectations from the outset so that all concerned this, you need to understand some aspects of the
can understand what a realistic outcome for that ways in which people function under these

214
CHAPTER 15 | COMMUNICATION, ORGANISATION AND LEADERSHIP IN SURGICAL CARE

circumstances, to learn to communicate better avoidance of leading and multiple questions;


and to think about the ways in which you can knowledge of and methods to overcome responses
contribute to the overall functioning and well-being such as denial and blocking; understanding and
of your team. Consider, for a moment, the range use of empathic statements (see glossary).
of tasks and scenarios you might face as a busy
surgical registrar on call over a weekend, where
TABLE 15.1
communication, leadership and team skills are
essential to a successful outcome. Having COMMUNICATION SKILLS AND
considered these skills, refer to Table 15.1 for
ORGANISATIONAL ISSUES TYPICALLY
a list of examples.
USED OR ENCOUNTERED DURING A
WEEKEND ON CALL
COMMUNICATION Some tasks/processes
Communication matters – not just with the – Informing patients and relatives of their
patient and relatives but also with colleagues. condition and progress
It is important to be able to communicate – Obtaining consent for interventions
effectively for many reasons: – Breaking bad news to patients or relatives
• to elicit and to provide information quickly – Arranging investigations and ensuring
and accurately. Especially in the critical care the results are seen and acted upon
setting, the quality and efficiency of this – Conducting ward rounds
aspect of communication (data transmission) – Speaking to senior colleagues
is important – Speaking to junior colleagues
• to be able to respond to psychological and – Booking cases for operating theatre and
emotional issues in colleagues, patients and ICU/HDU
relatives (empathy, understanding and emotional – Coping with multiple jobs
support). This may contribute to improved – Speaking to multiprofessional team members
outcome and will improve satisfaction ratings – Dealing with emergencies
and reduce complaints – Delegation and seeking help
• to be aware of the possibility of tension or – Identifying and treating patients
distress building up within the team and to know in clinical priority
how to respond to this. Your team will not Some skills
work well when it carries this type of burden. – Planning – Leadership
A comprehensive account of basic communication – Active listening – Mirroring
skills is outside the remit of this chapter, although – Empathy – Checking back
certain relevant communication skills are discussed – Assertiveness – Summarising
and practised on the CCrISP course. You should – Use of silent pauses – Prioritisation
develop an awareness of basic communication – Time management – Delegation
skills including: the appropriate use of open, – Avoiding aggressiveness
focused and closed questions; knowledge and

215
CARE OF THE CRITICALLY ILL SURGICAL PATIENT

WHAT ARE THE SPECIFIC into the overall management plan. It can be
COMMUNICATION PROBLEMS? helpful to try to predict what may happen and
Often, surgical critical care takes place in an have a plan for the different possibilities. With
environment where background obstacles to regard to patients, especially the elderly, if they
communication are more likely. The patients are are in an environment with limited natural light,
ill and frightened, and the staff are often over- this may increase disorientation. Readable clocks
busy. The patient may be unable to concentrate, or other ways of helping to overcome this are
especially if there is pain, severe illness or important. Especially where there is a degree of
complications of medication. Equally, operational organic confusion, aids to orientation can be
fatigue on the part of staff is also important. important, such as photographs of loved ones,
It is often easier for others to recognise the and easy-to-read name badges.
signs than for individuals to identify themselves. It may be necessary to repeat both questions
Signs of operational fatigue include loss of clinical and explanations at different times. Being prepared
sharpness and reduction in the quality of decision- to go back over the history after the immediate
making. Other obstacles to communication may crisis is good clinical practice and may reveal
include irritability and anger, high tension, issues previously unconsidered. It should be
confusion (most obvious in organic brain realised that many communication situations
syndromes but may also occur in functional are not single episodes but rather a continuous
disorders), distress and tearfulness, and high process involving multiple episodes over time.
expectations from patients and relatives but also Similarly, it is helpful to reduce fear by offering
from self and colleagues. repeated explanations and using check-backs to
assess that patient and relatives have understood.
SPECIFIC COMMUNICATIONS STRATEGIES Patients can often only recall only small amounts
The critical care setting of the information provided from a single
Strategies need to be targeted at the difficulties communication episode.
likely to be experienced. Critical care settings
can be bewildering for patients and their relatives – Breaking bad news
often with a lot of unfamiliar equipment, sometimes There is no perfect way to set out any
with limited access to natural light. It is easy to communication process – what works well with
assume that patients, relatives and doctors have some people in some situations can fail in others.
a greater knowledge and experience of these However, some general principles are probably
environments than they actually have. It is helpful. It can be useful to think about this is terms
especially important that at each stage, explanations of what educators call the ‘set’. This includes the
are provided. These can be very simple tasks, environment where the communication episode
such as explaining the role of a particular piece will take place and who will be present. It also
of equipment, an account of the next intervention includes an introduction as to the purpose of the
or an explanation of where a specific issue fits episode, the details of the episode itself and then
a summary of the salient points of the discussion.

216
CHAPTER 15 | COMMUNICATION, ORGANISATION AND LEADERSHIP IN SURGICAL CARE

When speaking to relatives, it is important to people find difficult to handle; once they know
ascertain that the patient has given permission what they are facing, they can start to deal with
for relatives to be informed of their condition. it and patients will often thank you for being
Understanding, if possible, intra-family dynamics frank and honest. Clearly, however, this can still
can also help manage communications with be a delicate situation.
relatives. For example, in some circumstances Attitudes have changed substantially in the last
it may be felt necessary for the medical staff to two decades but the work of John Hinton in the
talk to several family members together, while 1970s with people who had terminal illnesses
in others a family ‘spokesperson’ may be the best remains instructive. He found that, in an in-patient
person to communicate with. unit, though staff believed that only a small
In terms of breaking bad news, an important minority of patients knew of their diagnosis and
principle is to be prepared to talk, and listen. prognosis, a substantial majority had a very good
The barriers to doing this may come from understanding. This knowledge was acquired in
patients (or relatives if they are receiving the various ways, including overhearing bedside
communication) or directly from us. Some things conversations or reading case files. They were
are hard for us to talk about but, in this setting, able and willing to share this with Hinton in a
it is important to be able to tackle these. One way way that they had not done with the other staff.
of starting such a conversation is to ask an open Importantly, when asked why they did not discuss
question such as ‘what is your understanding of their knowledge with staff, patients often indicated
the present situation?’, or ‘what have you been that they did not want to cause the staff distress.
told so far?’. In this way, you are giving the In other words, patients chose silence partly to
patient or relative the first opportunity to have protect the staff working with them. From this,
a say and it may help you understand their the concept arose of being prepared and able to
expectations and how much they wish to be told. give the patient permission to talk about bad news.
Some patients want a lot of detail, others only a
To be able to give permission effectively requires
broad outline. If you are unaware of the patient’s
good listening skills. Listening is an active
expectations at the outset, you will not be able to
process, interspersed with signs of encouragement.
meet them and you should not make assumptions.
We all do this differently but should use attitude,
Starting in this way also gives you the opportunity
facial expression or verbal acknowledgements to
to show that you are listening and to pick up on
show interest and encourage further disclosure.
any verbal or physical clues as to the patient’s
or relative’s underlying emotions. These can be The use of empathic statements can be a straight-
subtle and you need to consciously look for forward way of identifying feelings and indicating
them. You need to be prepared to use direct and support. These are statements in which the
understandable language. It is a great temptation interviewer tries to identify a current feeling such
to ‘beat around the bush’ in an attempt to soften as sadness, anger or fear and then ties it to what
the blow but it is important to say difficult, has been happening, such as ‘It sounds as if this
emotive words such as ‘cancer’ or ‘death’, should news has made you feel more fearful than anything
they be appropriate. It is often uncertainty that else’. This can allow the person to talk about

217
CARE OF THE CRITICALLY ILL SURGICAL PATIENT

feelings and it also gives the interviewer a chance you should lean forward. It is not suggested that
to check if what he/she perceives is correct. There everything a patient does should be mirrored but
are different ways of responding to sadness, anger doing the opposite to what the patient is doing
and fear, for example. In contrast, sympathetic can send a message that you are not listening or
statements such as ‘I know just how you feel’, concerned about them.
should be avoided. It is very unlikely that you A further issue for more junior doctors in particular
could feel the same and such statements can lead is the way they handle their own uncertainties.
to aggressive reactions from patients or relatives. In general, patients want definite statements and
The most important aspects of helping people talk guarantees of outcomes. Clearly, there is much
about feelings are to allow time and space. The uncertainty surrounding surgical outcomes and
setting should be quiet and private. The interviewer you need to be able to appear confident in your
should give a sense of having time to talk. Often knowledge, yet not lead patients to have unrealistic
it will not take much time (in general, more skilled expectations.
communicators take less time than less skilled At the end of the discussion, it is useful to make
communicators) but it does require planning to it clear that further meetings can be arranged and
ensure, for example, that discussions like this are to give details of how this can be done. Giving
not started a few seconds before a ward round or the family a ‘liaison’ person can often provide
some other fixed event. There is evidence to show reassurance that it should be easy to talk again.
that if a person is left to talk freely that they will It is also important to document in the patient’s
speak for between 40–80 seconds. Allowing them notes that he or she has been spoken to, to provide
to do so will start things off on the right footing a brief outline of what was said and to record any
and help the patient think you care about their issues that may be relevant in the future.
problem. Sitting down to talk to the patient is
Medical mistakes
good not only from the body language point of
Occasionally, people come to harm following
view but gives the impression of more time being
a medical complication or a medical error. This
taken. In a study where a doctor, who was either
raises quite different communication issues.
sitting or standing, spoke to patients for a set
In addition to breaking bad news, there is the
length of time, the patients’ estimation of how
additional matter of handling guilt feelings and
long the doctor had spent with them was doubled
fear of litigation. Again, it is not possible to make
if the doctor had sat down.
absolute statements but, in general, even in these
Another aspect of communication to be aware situations, it is important to explain as fully as
of is the use of ‘mirroring’. The doctor mirrors possible and, if an error has been made, to offer
what the patient is doing in terms of their tone an apology. Not only is this in keeping with
and speed of speech, and their body language. current thinking in the NHS but, since a sense of
For example, if a patient is sounding timid and injustice often drives litigation, it is probably also
scared, using a similar tone may convince the a part of good risk management. It is important
patient that they are being listened to and dealt to be clear that one cannot apologise for the
with appropriately. If they are leaning forward, actions of others – you can state that you are

218
CHAPTER 15 | COMMUNICATION, ORGANISATION AND LEADERSHIP IN SURGICAL CARE

sorry to hear of their concerns/worries and that should also show respect in their own behaviour
they are entitled to a full reply to questions/ and learn how to use assertive rather than
complaints. It is dangerous to apologise for an aggressive or passive interaction (see glossary).
event that occurs outwith the individual doctor’s
control (i.e. by another doctor or member of staff).
It is also important to realise that you should EVERYDAY COMMUNICATION IN
not criticise actions of others without very careful SURGICAL CARE: ORGANISATIONAL
consideration. The GMC Good Doctor Guidelines SKILLS
stresses the importance of collegiality and it is Surgical training covers basic knowledge, operative
very easy to comment on something without skills and, through courses such as the CCrISP
knowing the full details. Criticism of others is course, guidance in practical management of
easy to imply by the most innocent off-hand acute conditions. Only infrequently do trainees
remarks or ill-guarded body language. In certain receive advice or instruction about the organisation
cases, such actions can give the patient the of their practice. Regrettably, this is often discovered
justification in their mind to make a complaint through trial and error.
or to seek legal advice.
In any training programme, there is a point
Working with colleagues where the surgical trainee begins to take increased
Staff relationships are of particular importance responsibility for hour-to-hour management of
in critical care settings. Not only does the work patients, for critical decision-making about the
involve vulnerable and dependent patients, it also requirements for treatment of emergencies, and
carries with it a lot of work-related emotional to carry out major and emergency operations as
issues. It is easy for these pressures to translate appropriate. This has historically been on promotion
into aggression and lack of respect. They may be to the registrar grade in the UK. Nowadays, the
made worse when inter-professional rivalries stage at which this occurs will vary but there is
intervene or when people normally outside the always a sizeable step-up in responsibility at some
unit are involved with particular patients. point in surgical training. Ultimate responsibility
rests with the consultant in charge but no surgeon
Ideally, there needs to be some way for these
can expect to carry out procedures without sharing
issues to be dealt with on a team basis – identifying
responsibility for peri-operative care.
problem areas and finding supportive and
effective ways of achieving change. Methods of Therefore, the senior trainee will often be
achieving this cannot be prescribed but must vary responsible for the daily business ward rounds,
with the precise situation. Deficient communication reporting as necessary to the consultant. It is
must be addressed, whether within or between unlikely that the consultant will make a formal
professional groups, either by individual or group ward round every day so it is essential that the
meetings, and formally or informally. These trainee actively manages the patients, looks for
techniques often remain alien to the medical and identifies problems, makes decisions about
profession but can help greatly in the development management and contacts the consultant when
of efficient and good-humoured units. Individuals appropriate. Initially, the trainee will communicate

219
CARE OF THE CRITICALLY ILL SURGICAL PATIENT

very frequently with his senior but, with training emergency theatre sister and a friendly radiologist
and experience, the scope for safe practice can are just some examples. Think about the people
and should expand. who can make things happen for your patient and
For care to be delivered successfully in the context for you.
of a busy surgical practice, there are many facets
of care during the working day including decision- ORGANISE TO MAKE COMMUNICATION
making, investigations and operations. These do EASY AND RAPID
not happen automatically! As the trainee, you To get the best out of a team, leadership is
have to learn to conduct this orchestra of activity, required – this requires a range of skills including
and it is not always an easy task. To achieve ability, knowledge, personability, decision-making,
success, you will need to organise yourself (and appropriate humour, humility, acceptance of other
sometimes others), exercise a degree of leadership, views and firmness. All must be deployed at the
communicate effectively and be able to make right time and few, if any of us, possess all or
appropriate decisions. even a majority of these attributes. You will need
To make decisions, you need information – and to work hard, praise and support your colleagues,
you get this from communication. As you train, admit when you are wrong or do not know and
you need to become aware of what information get timely help. Dealing with seniors is a whole
you do need and what information is largely skill in itself. Few consultants will not wish to
superfluous to any critical decision. There is a be informed promptly about unwell patients but
balance to be struck between hasty and unfounded all will expect you to have assessed the patient,
decision-making and un-necessary delay waiting begun immediate treatment and arrived at some
for tests that will add little or nothing. Getting decisions including a provisional plan of action.
information takes you or others time and you The exception to this is the patient who clearly
need to delegate and organise appropriately. needs an immediate operation beyond your
To be efficient, you need to time-table your ability – for example, a collapsed patient with
business ward rounds such that key information penetrating trauma – where the consultant will
is most likely to be readily available from nurses want a brief clear message and probably give
and house staff. You need to be prepared to you a brief and clear reply!
circumvent blocks to your patients’ progress. Clinically, you will need to lead by example –
It may prove difficult to get old notes, get a if you are not thorough, why will anyone else be?
certain test or opinion, administer a certain drug. Re-assessing patients is probably the single most
At times you will need to be quite assertive on neglected skill – clinical patterns will emerge and
your patients’ behalf to get what they need but diagnoses become obvious. With current working
you need to learn when to be, and when not to practices this is becoming more difficult and
be, assertive. Building up good personal working greater organisation is required for achievement.
relationships between other key members of staff This underpins one of the basic CCrISP principles,
will often help in this regard. The ward sister, of re-assessment after making decisions or
the out-reach nurse, a clinical nurse specialist, the instigating interventions. Utilise the support

220
CHAPTER 15 | COMMUNICATION, ORGANISATION AND LEADERSHIP IN SURGICAL CARE

services available – good and experienced nurses categories is invaluable in this situation. This
can give you an enormous amount, particularly should be supplemented by a verbal reinforcement
about how unwell a patient is. of which patients are giving cause for concern
and some acknowledgement from the doctor
Managing emergencies and deciding on the
need for urgent surgery is difficult. Patients need receiving the handover that these patients have
a diagnosis and unstable patients need treatment been identified and the responsibility accepted.
Assuming the incoming doctor will pick up on
urgently. Regrettably, patients do not magically
these things instinctively is not acceptable.
improve between 2 a.m. and the 8 a.m. ward
round – the emergency patient who fails to Progressing up the surgical training ladder is a
respond to simple resuscitation in the middle stressful but enormously rewarding time. You will
of the night needs a plan of action made then. very quickly develop new operative and patient
This may involve conservative or operative management skills and begin to feel that you
treatment but lack of knowledge or an inability really are a surgeon. Organise yourself and your
to conduct a particular operation is never an practice and communicate and listen effectively
adequate reason for delay. Decision-making is to make the learning process less stressful for all.
active not passive!
Continuity of care is essential for patient well- COPING WITH ADVERSE EVENTS
being in critical illness. Junior doctor hours have Emotionally charged events are common in
changed but the need has not – the onus is now everyday life and particularly so in the critical
on the ‘owning’ team to pass on problems to the care setting. This holds true for relatives and
duty team, but also on the duty team to look for staff as well as for patients. Coming to terms
problems among the patients of all the surgical with these everyday events is a largely automatic
teams and to deal with them promptly. This poses process. In simple terms, it seems to include
the challenging communication task of a surgical having an awareness of the emotional reaction
handover when the responsibility of a large and somehow returning towards a normal balance.
number of patients needs to be passed on to a Traumatic stressors are events that produce
different team. Within this patient group there intense pressure or tension; they are associated
will be patients who need specific interventions with the negative emotions of fear and sadness.
in a timely manner, patients who are getting better In normal circumstances, these emotional
and do not need anything specific, patients who reactions gradually decline and each subsequent
are at high risk of having problems, those who recall of these feelings is rather less intense until
have not yet been fully sorted out and those who eventually, as a new equilibrium is reached,
deteriorate unexpectedly. This latter group should the emotional reaction fades completely and the
be very small if the CCrISP principles of pro-active individual adapts.
management, particularly for those who are Faced with events that are perceived to be
‘slow’ or fail to progress, are adhered to. A written especially traumatic, this adaptive mechanism
handover list with a concise summary of each may be overwhelmed. The initial emotional
patient and a categorisation into one of the above reaction may be so intense that the only viable

221
CARE OF THE CRITICALLY ILL SURGICAL PATIENT

reaction is to attempt to prevent or avoid (blot to talk about a critical event, learning about some
out) these painful feelings. This may be achieved of the ways that people may respond and
by avoiding places or objects that remind the (usually) achieving an understanding that their
person about the trauma, or through suppression own behaviour is within a normal range.
of emotions in general – ‘emotional numbing’.
These defensive reactions will rarely be completely
COMMON PSYCHOLOGICAL DISORDERS
successful and the individual is left with painful
IN SURGICAL CRITICAL CARE
intrusive recollections, which alternate with
defensive avoidance. This cyclical reaction of So far, the emphasis has been on specific reactions
intrusion and avoidance is the central element of to adversity but of course a wide range of problems
post traumatic stress disorder (PTSD). It is possible may occur. Traumatic life events may trigger
that, as the emotions are suppressed because they feelings of depression, anxiety or even relapse of
are too extreme, they are not held in awareness certain psychoses. The assessment needs to cover
and do not decline. The condition becomes chronic the full range of psychological difficulties. In this
and may be frankly disabling. Stress disorders are section, brief reference will be made to four
not rare: some symptoms of PTSD are seen in the of these.
majority of patients who are involved in significant
accidents and features occur in relatives of the ANXIETY
victims and staff. Typically, patients may report Mild feelings of fear, apprehension, sadness and
recurrent and intrusive distressing recollections emotional turmoil are very common in anyone
of the event including flashback episodes. These admitted to hospital with a serious condition.
can be precipitated by cues, which symbolise or In general, the approach taken by the clinical
resemble an aspect of the traumatic event (e.g. team can often determine the amount of distress
hearing a car’s brakes on TV or even driving experienced. A team that works well together,
past the hospital). The victim is likely to avoid communicates well with patients and offers
thoughts or cues that activate memories of the appropriate emotional support will reduce these
event and may become withdrawn, detached or difficulties, while dysfunctional teams will
appear depressed. exacerbate the problem.
Critical events are a significant cause of Assessment is likely to centre on asking appropriate
occupational stress for staff groups (including questions about current feelings and enquiring
doctors) in this environment and this is important into any associated autonomic symptoms of
to recognise not only for personal and team anxiety (e.g. tachycardia, raised blood pressure)
well-being but also because operational fatigue which may mislead in the assessment of physical
and impaired performance may result. Awareness health. Sometimes, visible over-breathing
of stress reactions is the first step and the provision (excessive, often irregular breathing) may be a
of appropriate support of colleagues and patients, clue to the presence of the chronic hyperventilation
largely through opportunities for discussion, will syndrome. This can present with a multitude of
represent a significant advance in many settings. physical symptoms and is often associated with
The initial aim is to provide a means for people anxiety or depression.

222
CHAPTER 15 | COMMUNICATION, ORGANISATION AND LEADERSHIP IN SURGICAL CARE

Major depression problems) and can cause complications in the


Depression is also a common condition and often critical care setting. The characteristic problem
unrecognised. It spans a wide range of severities arises from withdrawal symptoms, which follow
and patterns of reaction. The core feature is a hospitalisation and enforced abstinence.
depressed mood, in which there is loss of pleasure These can include typical tremor, nausea, mood
and enjoyment, reduced interest, hopelessness and disturbances and confusion but may extend to
helplessness, and pessimism for the future. In delirium tremens and even convulsions.
addition, there are often biological features, such
as loss of weight, impaired sleep with early morning ACUTE ORGANIC REACTIONS
wakening and a diurnal variation of mood – Variously styled as confusional states, toxic
worst in the early morning. Finally, there may be confusional states, deliriums, etc., these are
evidence of a frank psychosis with mood-congruent short-lived organic disturbances characterised by
delusions and hallucinations. These may include confusion, clouding of consciousness (sometimes
delusions of worthlessness or guilt, delusions of quite subtle), disorientation and often marked
cancer, delusions of persecution (felt to be deserved), fearfulness. There may be delusions – often
accusatory auditory hallucination. All these are persecutory. Common causes include alcohol
in keeping with the primary disturbance of mood. withdrawal and prescribed medication
As a routine in the assessment of psychiatric (e.g. analgesia) but they may also occur in the
disturbance, there should be an investigation of context of a wide range of medical conditions.
suicidal thinking. One of the ways of asking about Following assessment and correction of the
this is to combine a permissions statement with ABCDs of the initial assessment, further evaluation
a question. For example, if someone has talked is centred on the cognitive state – ability to
about feeling very unhappy, they sometimes can attend and retrieve, awareness of environment,
not see much point in life. Then continue with etc., and on the possible causes which also require
something like ‘I wonder if you have ever felt it full assessment. This is an organic disorder in
would be better just to go to sleep and never wake the psychiatric classification because it is always
up?’ This can be followed by further questions secondary to some physical dysfunction. It is
about any suicidal thoughts, any suicidal plans likely to be made worse by a disorientating
(going into detail if needed) and any suicidal environment and by a failure to offer frequent
behaviour. In this way, the whole subject can be and repeated explanations.
covered easily without causing excessive concern.
There is no excuse for failing to ask about suicidal WHEN TO REFER TO A PSYCHIATRIST?
thinking in the presence of significant psychiatric To some degree, this depends on the capacity
disturbance. and engagement of the local psychiatric service.
However, there are clear indicators for referral,
ALCOHOL DEPENDENCE which are important to outline. First, there is the
This is included as a reminder that alcohol situation where the diagnosis is uncertain and
problems are common (in general, about 1 in 5 especially where there may be a psychiatric
people in hospital have significant alcohol-related component. Somatisation disorder and

223
CARE OF THE CRITICALLY ILL SURGICAL PATIENT

Munchausen’s syndrome are extreme examples SUMMARY


but there are often complex interactions between This chapter cannot provide a comprehensive
physical and psychological processes which may account of the field but perhaps it will help to
require assessment. It is important in these highlight those areas where further learning is
situations to make positive psychiatric assessments required. This learning is not readily available
rather than assumptions based on the absence of in text books but can be gained with experience.
signs of physical disorder. It does require insight and reflection on the part
There are situations where either the severity of the individual, the latter skill being easily
of the psychiatric condition or level of danger neglected in a busy surgical environment.
associated with the condition make referral both Communication skills are especially important
appropriate and often urgent. This might be as they help to make practice more effective and
following, for example, deliberate self-harm or efficient. More can be achieved in less time. It is
the development of persecutory beliefs in an acute important to look at patients, relatives and staff
organic reaction leading to thoughts of murder. groups and understand the ways in which we cope
with the everyday workload, with adversity and
One situation in which referral is often considered
how these mechanisms can be overwhelmed at
is in relation to consent. Psychiatrists have special
times of crisis.
knowledge of the legislation to do with consent
to treatment for psychiatric illness. The relevant
legislation has much less to say about consent to
treatment for physical illness and common law
principles usually apply. Nonetheless, as long as a
referral is not made with overoptimistic expectations,
it may still be useful to discuss difficult cases
where consent is withheld as this is an issue
which is more common in psychiatric practice.

224
CHAPTER 15 | COMMUNICATION, ORGANISATION AND LEADERSHIP IN SURGICAL CARE

GLOSSARY
Most clinicians could improve their communication A leading question expects a particular answer,
skills and surgeons are certainly no exception. e.g. ‘The pain is worst at night, isn’t it?’
The glossary outlines some principles about which Multiple questions include a list, e.g. ‘Do you
you may wish to read further. The specific skills have problems with chest pain, shortness of
cannot be summarised in a short glossary but are breath or ankle swelling?’ This might attract
included in most books on communication. the answer ‘no’ which to the patient might be
‘no’ to ankle swelling and to the doctor might
BASIC COMMUNICATION SKILLS be ‘no’ to the three items together.
In this section, some of the terminology will be
explained. It is useful in data gathering to use
There is a skill to checking back – being prepared
an appropriate range of open, focused and closed
to check that you have the right understanding –
questions. In taking a history, the open question
or using a summary of the main features as a way
‘Is there anything else?’ is useful as a final question.
of confirming the history with your patient.
There is also a skill to sharing a problem. If you
Open questions can take a wide range of do not know how to handle something in an
responses, e.g. ‘What is the main problem?’ interview, sometimes the best thing is to own up.
Focused questions can take a limited range of For example:
responses, e.g. ‘Which is the worst pain today?’
Closed questions must be answered ‘yes’ or
‘no’, e.g. ‘Is the pain in the knee the worst ‘I have a feeling that you are upset but I am
pain that you have?’ not sure what has caused it. Is it OK to ask you
about it?’

Some questions are likely to produce misleading ‘My problem is that I only have 5 minutes
answers. A leading question expects a particular before I have to go to theatre. I really need to
response and this may be given even if it is ask you about something. Is that alright?’
wrong. Multiple questions are common in
checklist approaches to the history but the answer Finally, perhaps the most useful of the active
given may only relate to the final item in the steps in understanding emotional reactions is the
list – again misleading. empathic comment. This is a statement identifying
an emotional reaction, e.g. ‘That must have made
you feel very frightened’.

225
CARE OF THE CRITICALLY ILL SURGICAL PATIENT

In making this statement, a lot of care must be ASSERTIVENESS, PASSIVITY AND AGGRESSION
exercised to listen to what is being said and not In being assertive, communication allows each
simply to assume that everyone will experience person to express their honest opinions without
fear, anger, sadness, etc. in specific situations. needlessly hurting the other person. In being
It is useful as a way of checking back on emotions passive, honest opinions are suppressed.
but, more importantly, it communicates that you
Aggression involves the use of excessive force
can appreciate at least some of what your patient
or power causing needless suffering. This can be
is feeling. This can be a very powerful intervention
active aggression (e.g. violent, insulting speech)
and should be a skill available to all doctors.
or passive aggression (e.g. emotional manipulation).

Blocking Assertiveness is, therefore, usually the preferred


Not facing an issue. For example, a patient asks option. In general, assertive statements contain
‘are there any complications with this operation’ the pronoun ‘I’ whereas aggressive statements
and the surgeon replies ‘don’t you worry, it’ll all more often include the pronoun ‘you’, for example,
be fine’. Another example is a doctor telling a ‘I feel that the patient would be better helped by
patient they have cancer and the patient says ‘it this approach’ versus ‘You are incompetent
can’t be cancer, I feel too well’. and have got this all wrong’.

Mirroring
Reflecting what the patient is saying in terms
of tone of voice and body language. For example,
if a patient is talking softly and timidly, reply
in similar tones. If a patient is sitting leaning
forward, do the same. Doing the opposite
(anti-mirroring) can adversely affect interactions.

226
16
Assessment of
surgical risk and
peri-operative care

227
CARE OF THE CRITICALLY ILL SURGICAL PATIENT

Many of the factors that increase surgical risk are


OBJECTIVES covered elsewhere in this book. This chapter will
discuss risk assessment in more detail and outline
This chapter will help you to understand:
the specific effects of older age, obesity and, in
• the importance of assessing peri-operative risk particular, diabetes.
• the factors that contribute to increased
surgical risk
• the peri-operative management of diabetes.
CO-MORBIDITIES AND
PERI-OPERATIVE CARE
Good surgical results reflect the quality of care.
Co-morbidity increases the risk of surgical This depends on:
procedures and minimising that risk is vitally • surgical factors, relating to pre-, intra- and
important to improve the individual outcome. postoperative care
Risk assessment is also important in terms of • patient factors, regarding disease presentation
outcome measures for comparative audit. and pre-existing co-morbidities
Simple scales, such as the American Society • systemic factors that relate to the resources
of Anesthesiologists (ASA) grading system, are available for the treatment of surgical patients.
open to varied interpretation among experienced Pre-existing co-morbidity increases the risk of
medical assessors, while more complex systems surgery. Anticipation of risk and risk factor
such as the POSSUM score, are too complex modification are vital in attempting to reduce
for most daily clinical applications. surgical morbidity and mortality. Co-morbidities
Co-existing diseases can complicate even a simple most commonly associated with increased
operation and increase morbidity and mortality. surgical morbidity and mortality are:
The level of care required needs to be anticipated • cardiovascular (hypertension, myocardial
with consideration given to transfer to units with ischaemia, cardiac failure and cardiac
appropriate facilities and/or to gaining disease- arrhythmias)
directed expertise to advise on pre-operative • chronic respiratory disorders
optimisation and peri-operative management of • anaemia
individual co-morbidities. • diabetes mellitus
The concept of a ‘high-risk’ patient is generally • chronic renal impairment
understood but the key is to recognise the factors • obesity.
contributing to that perceived risk and repeatedly Emergency surgical conditions entail higher
(re)assessing these patients throughout their stay risk as do elderly patients because of the higher
in hospital to minimise the risk of developing incidence of co-existing medical illness and
complications. a reduced physiological reserve.

228
CHAPTER 16 | ASSESSMENT OF SURGICAL RISK AND PERI-OPERATIVE CARE

THE METABOLIC RESPONSE TO INJURY the liver in an energy-consuming process, thereby


An understanding of the metabolic response to increasing hepatic oxygen consumption and blood
injury is helpful in understanding how co-morbidity, flow. Protein catabolism predominates principally
particularly diabetes, affects peri-operative affecting skeletal muscle but respiratory, gut and
management and risk (see also Chapter 13, (possibly) cardiac muscle are also affected, giving
Nutrition). Metabolic responses to major injury, rise to problems with mobility, ventilation and
surgery and severe infection have similar enteral nutrition. There are concomitant increases
mechanisms. The response occurs in two phases, in urinary excretion of nitrogen and creatinine.
referred to as ‘ebb and flow’. The mediators and The increase in proteolysis provides amino acids
their effects for these responses are outlined in as precursors for hepatic gluconeogenesis.
Table 16.1. The ‘ebb phase’ lasts 24–48 hours Concentrations of the amino acid glutamine in
and is a neuroendocrine response to tissue injury skeletal muscle fall. Glutamine is an important
and hypovolaemia. Cardiovascular reflex activity fuel for cells of the immune system and it is a
and inhibition of central thermoregulation are precursor for glutathione (a free radical scavenger);
reminders of the ‘fight or flight’ response. Energy it has a role in nitric oxide metabolism and has
stores are mobilised to fuel the increased metabolic also been implicated in the maintenance of the
demand: plasma glucose concentration increases gut mucosal barrier which may be compromised
in proportion to the severity of the injury due to after injury. Insulin resistance after injury refers
mobilisation of liver and skeletal muscle glycogen to its anabolic effects; for example, hepatic glucose
stores and the suppression of insulin release that production, lipolysis and the net efflux of amino
inhibits the uptake of glucose into cells. Lipolysis acids from skeletal muscle. These effects persist
is increased but fatty acid re-esterification within at plasma glucose and insulin concentrations that
adipose tissue may be stimulated by the raised are inhibitory in uninjured subjects. Uptake of
plasma lactate of severe injury or impaired glucose into skeletal muscle is also reduced, an
perfusion of fat deposits. An early rise in hepatic impairment that involves glucose storage rather
protein synthesis and an increase in microvascular than oxidation. The cause may result partly from
permeability are responsible for characteristic counter-regulatory hormones cortisol, adrenaline
changes in plasma protein concentrations within and glucagon, although infusion in healthy
6 hours. individuals requires much higher plasma
concentrations to cause insulin resistance than
Survival beyond the first 1–2-day initial phase
those found in injured or septic patients. So the
gives rise to the ‘flow phase’ of increased metabolic
effect of these hormones could be augmented
rate, principally due to muscle catabolism and
by modulation of insulin sensitivity by
resistance to the anabolic effects of insulin.
pro-inflammatory cytokines: interleukin-6 in
The triggers for this are similar to the first phase
cancer patients, interleukin-1 in endotoxaemia
but with increased energy consumption. The high
and tumour necrosis factor in diabetes and obesity
energy source ATP is produced principally by
are all correlated with the degree of insulin
glycolysis (an inefficient mechanism) and the
resistance.
lactate produced is reconverted into glucose in

229
CARE OF THE CRITICALLY ILL SURGICAL PATIENT

DIABETES MELLITUS
TABLE 16.1
Diabetes is a state of impaired glucose tolerance
MEDIATORS OF INJURY RESPONSE caused either by absolute lack of insulin (type 1)
AND THEIR EFFECTS or relative lack of insulin (type 2). In addition to
the metabolic disturbance, micro- and
Counter-regulatory hormones
macrovascular abnormalities cause retinopathy,
(e.g. catecholamines [adrenaline], cortisol,
nephropathy, neuropathy, coronary heart disease,
glucagon, antidiuretic hormone)
stroke and peripheral vascular disease. Diabetics
– Breakdown of glycogen stores in liver
also develop cataracts and specific soft tissue
and skeletal muscle
disorders such as diabetic cheiroarthropathy as
– Suppression of insulin release resulting
a result of exposure of the tissues to
in reduced uptake and oxidation of glucose
hyperglycaemia, causing accelerated irreversible
Increased sympathetic nervous system activity biochemical and structural changes normally
– Lipolysis found in ageing. Improved glycaemic control in
Protein metabolism diabetes protects against these secondary effects.
– Increased hepatic synthesis (interleukin-6 Surgery can be hazardous to diabetic patients.
induced) The metabolic response to surgical trauma can
– Increased microvascular permeability rapidly lead to hyperglycaemia and ketoacidosis,
– Raised plasma concentration of fibrinogen especially in insulin-deficient patients. Poorly
and C-reactive protein controlled diabetes accelerates catabolism and
– Fall in plasma albumin concentration delays healing. Insulin and the sulphonylureas
Pro-inflammatory cytokines (e.g. tumour can cause severe hypoglycaemia in fasted and
necrosis factor-, interleukins ß, 2, 6 and 8 anorexic patients, which can be particularly
– Mimic some responses, but plasma levels dangerous during general anaesthesia.
not universally linked to injury indicating Assessment of fitness for surgery, pre-operative
autocrine/paracrine (cf. endocrine) function optimisation, an agreed management policy
Interleukin 6 induction of prostanoids between specialists and ward staff and meticulous
at the blood–brain barrier glycaemic control will greatly reduce the risks of
– Activation of the operating on diabetic patients.
hypothalamus–pituitary–adrenal axis Some case examples will serve to illustrate the
key management issues in patients with diabetes
undergoing surgical treatment.

230
CHAPTER 16 | ASSESSMENT OF SURGICAL RISK AND PERI-OPERATIVE CARE

CASE SCENARIO 16.1


A 33 year-old civil servant, admitted to the surgical unit for an open elective inguinal herniorrhaphy,
is stabilised on thrice daily subcutaneous soluble insulin injection and is otherwise fit and well.
HOW WOULD HIS DIABETES BE MANAGED PERI-OPERATIVELY?
Such patients have absolute deficiency of insulin and require an intravenous glucose-potassium-
insulin (GKI) infusion to be instituted on the morning of surgery after an overnight fast and omission
of the morning insulin dose. A protocol for GKI infusion is given in Table 16.2, though note that
most hospitals will have their own local protocols.
He had an uneventful procedure under general anaesthesia on the morning list with hourly
BM recordings showing good control with plasma levels of glucose between 5–10 mmol/l. Due to
postoperative nausea and vomiting, the GKI regimen was continued overnight. By the next morning,
he was able to eat and drink normally; the GKI infusion was discontinued and he had his normal
morning insulin dose with his breakfast and was discharged home later that morning.

CASE SCENARIO 16.2


A 67-year old retired coach driver was seen in the pre-admission clinic for work-up for an elective
TURP for symptomatic benign prostatomagaly. He had been diabetic for 5 years and currently
controlled with diet and glibenclamide.
HOW WOULD YOU MANAGE HIS DIABETES?
Generally, type 2 patients well-controlled by diet or oral agents may simply omit their oral agents
and breakfast on the morning of surgery. However, long-acting sulphonylurea drugs (e.g. glibenclamide)
should be substituted by short-acting ones (e.g. glicazide) some days before surgery to reduce the
risk of hypoglycaemia. Blood glucose should be monitored closely in the peri-operative period and
persistent hyperglycaemia should be treated with a GKI infusion. If the patient is in a steady state,
the GKI infusion will maintain satisfactory glycaemic control and prevent hypokalaemia. GKI bags
must be changed if glucose levels are unsatisfactory. Alternatively, insulin may be given as a variable
rate intravenous infusion according to a sliding scale (Table 16.3), which provides greater flexibility.

231
CARE OF THE CRITICALLY ILL SURGICAL PATIENT

TABLE 16.2 TABLE 16.3

PROTOCOL FOR GLUCOSE-POTASSIUM- AN EXAMPLE OF SLIDING SCALE


INSULIN (GKI) INFUSION INSULIN INFUSION
For stabilised patients only This is for intravenous use in unstable patients
For intravenous infusion use 500 ml of 10% frequently with hyperglycaemia. 50 U of
dextrose containing 15 U of soluble insulin soluble insulin is added to 50 ml of 0.9%
(e.g. Actrapid) and 10 mmol of KCl isotonic saline giving a 1 U/ml solution for
• infuse at 100 ml/h delivery using a syringe driver
• check blood glucose every hour • hourly monitoring of blood glucose
• check K+ every 6 h and adjust accordingly concentration is mandatory
If blood glucose falls to < 5 mmol/l discard • start infusion at 6 U/h until the glucose
existing bag and replace with 10 U of soluble level begins to fall
insulin and 10 mmol of KCl • subsequently, titrate so that blood glucose
falls by 3–4 mmol/l each hour
If blood glucose rises to > 10 mmol/l discard
• most patients need 1–3 U/h and this
existing bag and replace with 20 U of soluble
usually becomes clear after 3–4 hours of
insulin and 10 mmol/l KCl
glucose monitoring
Lower or higher insulin doses are sometimes
A typical sliding scale would be:
needed
• blood glucose < 5 mmol/l, give 0.5 ml/h
Check plasma K+ every 6 h and adjust • blood glucose 5–10 mmol/l, give 2.0 ml/h
accordingly • blood glucose > 10 mmol/l, give 4.0 ml/h.
This regimen is only for patients with stable
and well-controlled diabetes; it should be
started on the morning of surgery after an
overnight fast and continued until the patient
can eat and drink normally

232
CHAPTER 16 | ASSESSMENT OF SURGICAL RISK AND PERI-OPERATIVE CARE

CASE SCENARIO 16.3


A 66-year-old woman was admitted from A&E with a 6-day history of feeling unwell, with immobility
and discharge from the right foot. She had been diabetic for 7 years treated with diet, glicazide and
metformin. Examination revealed discharge from the instep, a necrotic heel and cellulitis extending
across the ankle into the lower leg. Her pulses were all palpable, but there was diminished pedal
sensation. Her Hb was 11.1 g/dl, white count 18.6 x 109/l, blood glucose 24 mmol/l, potassium 5.8
mmol/l, urea 9.0 mmol/l and creatinine normal.
HOW WOULD YOU MANAGE THIS PATIENT?
She should be managed as per the CCrISP protocol, with immediate attention to the ABCs. Her diabetes
is out of control. An intravenous insulin infusion according to a sliding scale (Table 16.3), with hourly
monitoring of the blood glucose and 3–4-hourly potassium estimation (Table 16.4) is appropriate.
She requires resuscitation with intravenous 0.9% saline until she has been stabilised, and her glycaemic
and potassium control optimised. Administration of broad-spectrum or ‘best-guess’ intravenous
antibiotics aid stabilisation of sepsis and the metabolic state prior to definitive treatment of foot
sepsis by debridement or amputation will give the best result. Rarely, gas-forming organisms may
be present; if gas gangrene is suspected, urgent surgery will be required after initial resuscitation.

CASE SCENARIO 16.4


A 68-year-old, overweight woman was admitted in a coma. Her family provided a history of abdominal
pain, anorexia and vomiting for 1 week against a background of 15 years of diabetes mellitus. In the
early years, her diabetes was controlled by diet and oral hypoglycaemic agents; however, for the past
8 years, she had required subcutaneous insulin supplementation. She appeared to be dry and had a
temperature of 38.3˚C, a pulse of 130/min and systolic blood pressure of 80 mmHg. There was
epigastric fullness and guarding and she had sighing respiration with a smell of acetone on her
breath. A chest X-ray showed basal atalectasis. Blood results showed Hb 10.1 g/dl, WCC 19.5 x 109/l,
Na 152 mmol/l, K 6.7 mmol/l, HCO 15 mmol/l, Cl 100 mmol/l, Ur 22.5 mmol/l, Cr 85mmol/l, glucose
36 mmol/l. Her urine was strongly positive for ketones.
WHAT IS THE MANAGEMENT OF THIS LADY?
Clearly, this is a complex clinical problem that cannot be managed in a general ward. However,
again, the patient should be managed according to the CCrISP protocol with simultaneous immediate
assessment and resuscitation. Although she probably has a surgical problem, she also has diabetic
ketoacidosis (DKA) and this needs to be managed jointly with endocrinologists in an HDU or ICU,
with continuous ECG monitoring and close biochemical monitoring of glucose (hourly) and potassium
(3–4 hourly) levels (Tables 16.3 and 16.4).

233
CARE OF THE CRITICALLY ILL SURGICAL PATIENT

tissues including the brain; they provide a


TABLE 16.4 few percent of the total energy needs after
an overnight fast but this rises to one-third in
A POTASSIUM REPLACEMENT REGIMEN
prolonged fasting. When produced in excess,
In an unstable patient with varying insulin they accumulate rapidly as uptake mechanisms
requirements, serum potassium levels should become saturated. The main consequences of
be monitored every 3–4 hours. raised circulating ketones are:
Replacement should be guided by the latest • acidosis, both extracellular and intracellular
serum K+ concentration • diuresis, as osmotically active ketones are
• if K+ > 5.0 mmol/l, omit KCl due to the filtered in the urine, exacerbating the osmotic
risk of cardiac arrhythmias diuresis caused by glycosuria, and resulting
• if K+ 3.5–5.0 mmol/l, add 20 mmol KCl in polyuria, electrolyte losses, dehydration
to each litre of intravenous fluid and hypovolaemia
• if K+ < 3.5 mmol/l, add 40 mmol KCl to • nausea – by direct stimulation of the
each litre of intravenous fluid chemoreceptor trigger zone in the medulla.
Simultaneous resuscitation and investigation
includes a 12-lead ECG, bacteriological cultures
DKA is uncontrolled hyperglycaemia with of all appropriate fluids including blood and
hyperketonaemia severe enough to cause metabolic urine, cardiac enzyme determination and ABG
acidosis. It is caused by severe insulin deficiency analysis. Intravenous saline and insulin should
that stimulates lipolysis and a massive increase begin immediately (Table 16.3). Urgent treatment
in ketogenesis. It is the hallmark of poorly treated with scrupulous clinical and biochemical
type 1 diabetes but can occur in type 2 diabetes monitoring is essential. Correction of hypovolaemia
when patients are relatively insulin deficient and will often improve acidosis and hyperglycaemia.
there is intercurrent illness stimulating counter However, over energetic fluid and insulin
regulatory hormone secretion (especially replacement can predispose to cerebral oedema
glucagon). It carries a mortality of 5–10% (50% and increase mortality. CVP monitoring for the
in elderly patients with DKA precipitated by elderly and those at risk of heart failure is
myocardial infarction or infection). Prompt diagnosis required. Monitoring for response is essential.
and management is essential to prevent death. 0.9% saline (with K when appropriate, see Table
As well as hyperglycaemia, hyperketonaemia 16.4) is the fluid of choice. Dextrose 5% is
occurs due to oxidation of free fatty acids (FFAs) substituted when plasma glucose has fallen to
in hepatocyte mitochondria (a process stimulated 10–14 mmol/l to prevent hypoglycaemia (insulin
by glucagon and inhibited powerfully by insulin), is still required to prevent ketogenesis and
yielding ATP and acetyl-CoA. The latter is promote glucose utilisation in the tissues).
converted to acetoacetate, which may be oxidised Use of bicarbonate and hypotonic solutions is
to 3-hydroxybutyrate or undergo condensation to contentious; hypotonic solutions may exacerbate
produce acetone. Ketones are transported out of intracellular movement of water and could lead to
the liver and used as metabolic fuels by various cerebral oedema, while bicarbonate may improve

234
CHAPTER 16 | ASSESSMENT OF SURGICAL RISK AND PERI-OPERATIVE CARE

extracellular acidosis; however, as membranes HYPOGLYCAEMIA


are not permeable to HCO3– ions, the all important Hypoglycaemia is a dangerous side-effect of
intracellular acidosis may not be improved. Indeed, drugs that raise circulating insulin (i.e. insulin
CO2 can enter the cells to combine with H2O and sulphonylureas). It does not occur with dietary
to produce H2CO3 that can worsen intracellular restriction, metformin or thiazolidinediones (e.g.
acidosis with an adverse impact on outcome. roziglitazone). Common factors that predispose
Hyperosmolar non-ketotic (HONK) state is to hypoglycaemia are outlined in Table 16.5.
distinguished from DKA by the absence of gross
ketonaemia and metabolic acidosis. Hyperglycaemia
TABLE 16.5
may rise to higher levels but insulin levels are
high enough to suppress ketogenesis. It is found
COMMON FACTORS CONTRIBUTING
in previously undiagnosed type 2 diabetes and
TO HYPOGLYCAEMIA
may be precipitated by intercurrent illness,
diabetogenic drugs (corticosteroids and thiazide Accelerated insulin absorption
diuretics) or fizzy glucose drinks. Complications – Exercise
include thrombotic events such as CVA, – Hot environmental conditions
peripheral arterial occlusion, DVT and pulmonary Unfavourable factors relating to insulin
embolism, due to increased blood viscosity. administration
Mortality exceeds 30% because these patients are – Too early
old and often have a serious precipitating illness. – Too much
Lactic acidosis is generated rapidly during tissue – Inadequate food intake
anoxia (e.g. shock, cardiac failure or pneumonia)
Alcohol consumption
or when liver gluconeogenesis is impaired.
– Inhibits hepatic gluconeogenesis
In diabetes mellitus, it is a rare, but fatal,
Impaired hypoglycaemic awareness
complication of biguanides (phenformin, metformin),
which act by inhibiting gluconeogenesis. It presents Diabetes control too ‘tight’
as coma with metabolic acidosis and a wide Weight loss
amino gap due to hyperlactataemia. Blood glucose
Loss of counter-regulatory hormones
levels are usually raised. Treatment is difficult:
– Addison’s disease
intravenous bicarbonate may aggravate intracellular
– Hypothyroidism
acidosis; forced ventilation to blow off CO2 may
– Hypopituitarism
help; dialysis clears lactate and H+ and will
– Blunted glucagon secretion (as in
correct any sodium overload from bicarbonate
long-standing type 1 diabetes)
infusion. Mortality is high (> 30%) because of
– Intestinal malabsorption
co-existing organ failures.
– Renal failure (impaired insulin clearance)

235
CARE OF THE CRITICALLY ILL SURGICAL PATIENT

The events as blood glucose falls are listed in Obesity increases the likelihood of associated
Table 16.6 but without early recognition can medical disorders including ischaemic heart
precipitate a coma. Hypoglycaemia should be disease (especially central obesity), hypertension,
recognised and treated immediately. oesophageal reflux, diabetes, obstructive sleep
apnoea, osteoarthrosis, gallstones, varicose veins
and haemorrhoids.
PRACTICE POINT Reaching a diagnosis is often rendered more
Always remember to check the blood glucose difficult. General anaesthesia and surgical
with a BM stick in any patient with a reduced procedures are more hazardous and postoperative
level of consciousness. complications, especially those relating to
cardiopulmonary events, venous thrombo-embolism
and the wound, are more frequent.
For elective surgery in non-life threatening
TABLE 16.6 conditions, pre-operative weight loss should be
recommended. For all operations, a minimum
CLINICAL EVENTS AS BLOOD
of a blood glucose and ECG should be checked
GLUCOSE FALLS
pre-operatively. Further investigations and
~3.8 mmol/l Adrenaline and glucagon pre-operative optimisation will depend on other
secretion increases patient and surgical factors. Proceeding to
~3.0 mmol/l Onset of hypoglycaemic elective surgery requires a balance of risk versus
symptoms (note, hypoglycaemic benefit and may require careful discussion with
unawareness in some patients) the patient.

~2.8 mmol/l Neuroglycopenia and cognitive


impairment ELDERLY
< 1.0 mmol/l Coma A social definition of elderly includes all those
over 65 years of age and this group comprises
about 1 in 4 patients admitted to surgical wards.
OBESITY Increasingly, patients over 80 years of age are
Excessive body weight is an increasing problem being considered for major surgery and these
in people of all ages in the UK. The reference scale patients provide a special challenge.
for obesity is the BMI, where the BMI is given by Two main reasons for increased risk with ageing
weight (in kg)/height (in m2). are the frequent association of age and concurrent
The normal range for BMI is 20–25 kg/m2. medical problems and decreasing functional
Obesity is defined as a greater than 20% increase reserve in many organ systems making the
over the ideal body weight, which equates to a elderly less able to respond to the physiological
BMI over 30 kg/m2. consequences of operation. This is especially true
for the respiratory system, cardiovascular system,

236
CHAPTER 16 | ASSESSMENT OF SURGICAL RISK AND PERI-OPERATIVE CARE

kidneys, nervous system and drug handling. Again, In the work place, mortality and morbidity
careful assessment, optimisation and peri-operative conferences provide a forum in which factors
care should reduce the surgical risk. that have contributed to adverse outcomes can
be debated and strategies may be developed to
improve unit outcomes. National surveys such
RISK ASSESSMENT as NCEPOD (National Confidential Enquiry into
Defining levels of risk to patients is important, Patient Outcome and Death) allow panels of experts
both for enhancing the outcome of surgical to analyse surgical deaths and make conclusions
intervention, and for managing the expectations about their causes and recommendations for
of patients, their relatives and our colleagues. prevention. This can be around pre-operative
Verbal and written communication are vital to preparation, the grade and seniority of staff
the management of expectation in these groups involved and the resources available for treatment
because there is a general perception that poor (e.g. NCEPOD theatres, ICU and HDU).
communication by surgeons may hide poor Measurement of risk aims to provide some
performance. objective evaluation of individual patient risk and
can allow comparison of individual clinicians or
Assessment of clinical risk is a complex higher
units. This is fertile soil for research and a recent
function to which all doctors aspire and which
Medline search for surgical risk scoring revealed
forms an integral part of training. Apart from
almost 1500 publications. Thus scoring systems
direct clinical experience, how can we improve
have been developed in most subspecialties and
our risk assessment?
for many individual conditions or procedures to
Evidence-based medicine provides different levels try and produce a scale that will allow an accurate
of confidence about the outcome of an intervention prediction of outcomes for each patient. Highly
when examining published results. The most robust complex scoring systems may be unwieldy in the
evidence comes from randomised controlled trials clinical situation and, when found to be valid in
(RCTs) and meta-analyses of several RCTs on the one unit or specialty, may require modification
same topic, while case-controlled series provide for successful adaptation to other specialties
lower levels of evidence and case reports provide (e.g. the physiological and operative severity
the lowest (but not always insignificant) form of score for enumeration of mortality and morbidity,
evidence. These data often suffer from the constraints or POSSUM). Simple and more widely applicable
of carefully conducted trials but can be used in scoring systems such as the ASA grading system
one’s own practice to establish criteria for audit. is simple and in wide clinical use (Table 16.7).
Audit aims to improve the care of patients by Most patients will be assigned an ASA grade
establishing a standard, identification of areas for (1–5) by the anaesthetist assessing the patient pre-
improvement and implementing that improvement, operatively. Although simple and widely used, it is
then evaluating the effects of implementation. open to individual variation and even experienced
Audit can also be national with contribution of anaesthetists may vary in their assessment of the
data to national databases that are being established same patient. This blunts its sensitivity and ability
by the surgical specialty associations. to discern actual risk for each patient.

237
CARE OF THE CRITICALLY ILL SURGICAL PATIENT

TABLE 16.7

THE ASA SYSTEM FOR GRADING SURGICAL RISK

Grade Definition Mortality (%)


I Normal healthy individual 0.05
II Mild systemic disease, does not limit activity 0.4
III Severe systemic disease, limits activity but is not incapacitating 4.5
IV Incapacitating systemic disease, constantly life-threatening 25
V Moribund, not expected to survive 24 h with or without surgery 50

Risk management is developing in healthcare An international campaign to reduce harm in


and is borrowing ideas and techniques from the peri-operative care has led to initiatives such as
aviation industry that has practised risk avoidance ‘Patient Safety First’ within the NHS. This is based
with great success over many years. In addition on an acknowledgement by healthcare workers
to anticipating risk and the development of that events that produce harm in patients are
appropriate preventive strategies, a cultural change potentially avoidable. An example of one such
around the use of the information is required. initiative is that of reducing surgical site infections
Using adverse incident monitoring as an education by establishing a target reduction and examining
tool, rather than one for apportioning blame, compliance with appropriate interventions to
allows for learning appropriate lessons and putting achieve it, including appropriate use of prophylactic
strategies in place that minimise the likelihood antibiotics, maintaining normothermia, maintaining
of repeated failure. Also, aviation simulators that glycaemic control in diabetics and using
reproduce critical incidents allow important skills recommended hair removal methods. In addition,
to be developed that can then be put into practice to increase safety in the operating theatre, the use
at appropriate moments when real lives are at of the World Health Organization Surgical Safety
stake. In surgery, this translates to the use of Checklist is being piloted in hospitals. This is a
courses like CCrISP and skills laboratories in which ‘time-out’ before surgery (degree of urgency
clinical skills and techniques can be practised in permitting); at the end of the procedure prior to
the context of simulated patients and procedures transfer to the recovery area, the anaesthetist,
in order that best practice can be learned and surgeon, scrub nurse and other theatre staff
honed, for use in the clinical arena. discuss the patient preparation and any anticipated
critical moments or potential complications and
how these will be managed.

238
CHAPTER 16 | ASSESSMENT OF SURGICAL RISK AND PERI-OPERATIVE CARE

As champions for risk management, surgeons can


demonstrate leadership in the care of patients that
will improve the outcome of our treatment and
operations.

SUMMARY
• it is essential to recognise the factors that
contribute to surgical risk
• ensure patients are as fit as possible prior
to surgery
• be aware of co-morbidities to predict and
prevent peri-operative problems
• manage diabetes carefully and precisely in
the peri-operative period.

239
CARE OF THE CRITICALLY ILL SURGICAL PATIENT

INDEX adenosine 72, 74


adrenal failure 93
ABC approach 26, 35 adrenaline 31, 118, 119
ABCDE 13–17 endogenous 229, 230
abdomen -adrenoceptor agonists 118
assessment on ICI 140–1 ß-adrenoceptor agonists 118
burst 148 adult respiratory distress syndrome (ARDS)
compartment syndrome 142–3, 145–6 49, 177, 178
penetrating trauma 139 adverse events, coping with 221–2
re-operative surgery 140, 141, 151 adverse incident monitoring 238
sepsis 151, 171, 175, 176, 178 adverse outcomes
abdominal aortic aneurysm, ruptured 139 communication problems 214
abdominal compartment syndrome 142–3, 145–6 prevention 3
ABG, see arterial blood gases age
ACE inhibitors 87, 130, 160 and surgical risk 236–7
acid–base balance 52, 55–6, 103–4 see also elderly patients
aetiology of disturbances 56 aggressive communication 226
examples of disturbance 57–8 air bronchogram 41
management of disturbances 56 airway adjuncts 14, 27, 28
acidosis airway management 13–14, 26–7
DKA 234–5 escalating need 27
metabolic 55, 56, 57, 58, 104 golden rules 26
respiratory 55, 56, 104 oxygen masks 26–7, 42
activated protein C 177 techniques 26–7
acute coronary syndromes 82 tracheostomy 28–32
Acute Kidney Injury (AKI) classification 125 see also ventilation
acute organic reactions 223 airway obstruction, episodic 200–1
acute renal failure (ARF) 122, 125–8 alanine 192
aetiologies 125–6 albumin
case scenarios 124, 127, 134 intravenous 189
complications 132–3 serum 100, 194–5, 230
definition 125 alcohol dependence 223
incidence 125 aldosterone 93, 160
investigations 128–9 alkalosis
management 130–1 metabolic 56, 58
mortality 122 respiratory 55, 56
patient assessment 128 Allen’s test 52, 109, 109
prognosis and recovery 133 alteplase (rTPA) 81
acute tubular necrosis (ATN) 126, 129, 133 amino acids 185, 192, 229

240
INDEX

aminoglycosides 130 arterial line


amiodarone 76 ABG samples 52
anaemia 36 management 108
anaesthetist 14, 27, 98 signal transduction 108
analgesic agents 205–7 arterial pressure monitoring 109–11
analgesic ladder (WHO) 204, 205 arterial waveform 111
anaphylaxis 93, 100 aseptic techniques 154, 191
anastomotic leakage assertive communication 226
colonic 141, 150–1, 172 atelectasis 39, 48
diagnosis 151 atrial fibrillation (AF) 72, 73, 75–6
risk factors 151 treatment 76
signs of 150 atrial flutter 76
urinary 165–6 atropine 79
angina, unstable 82 audit 237
angiotensin–aldosterone 157 autonomic manoeuvres 72
anion gap 54 aviation industry 238
anti-diuretic hormone (ADH) 157 AVPU system 17
anti-emetics 207
antibiotics bacteriological samples 99
diarrhoea 189 bag/valve/mask system 27, 28
prophylactic 175, 181 ‘balance salt solution’ 163, 164
sepsis 173, 175, 177 barotrauma 45, 46
anticoagulation base deficit 54
and epidural analgesia 209 base excess 54
MI 81 ß-blockers 62, 81, 95, 130
pulmonary embolism 49 beta-2 agonists 37, 38, 132
anuria 128, 145 bicarbonate
definition 123 acid–base balance 55, 56
anxiety, patient 222 infusions 130, 132, 234–5
appendicitis 139 normal plasma levels 54
arrhythmias, common causes 74 biguanides 235
arterial blood gases (ABG) 44, 203 bisphosphonate 161
importance in surgical patient 52 bladder pressure, assessment 146
interpretation 53–4 blood cultures 173
respiratory failure 36 blood gases, see arterial blood gases
samples 52 blood loss
classification 92
see also haemorrhage

241
CARE OF THE CRITICALLY ILL SURGICAL PATIENT

blood pressure 62 C-reactive protein 129


central venous (CVP) 111–15 calcium 160–1
effect of haemorrhage 95 calcium gluconate 132
elderly patient 62 calcium phosphate 129
non-invasive measurement 107 calcium resonium 133
peripheral arterial 109–11 capillary permeability 92, 97
shock 90, 95 capillary refill time 94
septic 96, 97 carbohydrates 185
see also hypertension; hypotension carbon dioxide, partial pressure (PaCO2) 34, 43, 56
blood tests normal range 54
nutritional assessment 194–5 pain management 206–7, 208
postoperative complications 141 carboxyhaemoglobin 36
renal failure 128, 129 cardiac enzymes 65
blood transfusion 16 cardiac failure 42, 82–6
body language 218 acute management 86
body mass index (BMI) cardiac filling pressure 95
normal range 236 cardiac index (CI) 96, 119
and nutritional status 184, 194 cardiac output
obesity 236 abdominal compartment syndrome 145
bowel infarction 139 decreased 65, 92, 118, 145
brachial artery cannulation 109 determinants 106
bradyarrhythmias 79 drugs increasing 118, 119
bradycardia 31, 79, 93 measurement 116–18
brain, energy demands 192–3 in shock 96
brain natriuretic peptide (BNP) 65 cardiac silhouette, enlarged 41–2
breaking bad news 216–18 cardiac tamponade 96
breakthough pain 199, 211 cardiogenic shock 86, 91, 92, 95, 97
breath sounds 64 causes 95
breathing features 95
assessment and treatment 14–15 management 97
and pain management 201 cardiovascular disease (CVD) 60
see also respiration; respiratory failure acute coronary syndromes 82
bronchitis 41 atrial fibrillation 75–6
bronchoalveolar lavage (BAL) 37 atrial flutter 76
bupivacaine 209, 210 bradyarrhythmias 79
burns 92 case scenarios 73, 85
burst abdomen 148 congestive cardiac failure 42, 82–6
business ward rounds 8, 18, 20, 219, 220 hypertension 87
hypotension 71–2

242
INDEX

left bundle branch block 78 shock 93, 101


left ventricular hypertrophy 77 surgical site complications 142, 147, 149
management plan 71 CCrISP assessment system 12, 24
in MOF 177, 179 CCrISP course, aims 3, 9
myocardial infarction 79–81 central vein cannulation 112–13
right bundle branch block 78–9 complications 98, 115, 191
right ventricular hypertrophy 77–8 fluid replacement 114
risks of surgery 86–7 parenteral nutrition 190–1
signs 61 central venous pressure (CVP) 62, 95
tachyarrhythmias 72–5 abnormalities 17, 63
cardiovascular monitoring and intravascular volume 115
arterial pressure 109–11 measurement 98, 99, 111–15
blood pressure 107 pitfalls 115
cardiac output/index measurement 116–18 sepsis 171
central venous pressure 107, 111–15 shock 98, 103
clinical 106 waveform 114
indications 107 cerebral hypoperfusion 61, 94, 99
shock 102, 103 chart review 18–19
cardiovascular support 118–19 cardiovascular disease 61–2, 64
cardiovascular system pain management 202
effects of abdominal compartment syndrome respiratory problems 36
145 sepsis 171
examination 64–5 shock 94
cardioversion 73, 74, 76 see also drug charts; fluid balance charts
carotid sinus massage 72, 74 chest drain 50, 63
case notes 22, 23, 64 chest examination 15
examination 19 chest X-ray
operation records 138–9, 144 air bronchogram 41
case scenarios ARDS 177
acid-base disturbance 57–8 atelectasis 39
cardiovascular disorders 73, 85 bronchitis and emphysema 41
diabetes mellitus 231, 233 cardiovascular disease 42, 65–6, 84
fluid management 165–6 chest drain 50
pain management 199, 208, 210 consolidation 41
patient assessment 19, 22 interpretation 40–2
prevention of complications 4, 7 Kerley B lines 41, 42
renal failure 124, 127, 134 pericardial effusion 41–2
respiratory failure 38, 43 pleural effusion 41
sepsis and MOF 172, 174, 178–9, 180 renal impairment 128, 133

243
CARE OF THE CRITICALLY ILL SURGICAL PATIENT

respiratory compromise 37, 41–2 confusion 14, 61, 94


value of 40 conscious level 14, 61, 64
chloride depletion 56 cardiovascular disorders 61, 64
chloride ions 164 causes of alteration 17
chronic obstructive pulmonary disease (COPD) 53 opioid analgesia 202, 207
chronic renal failure 129, 135 respiratory failure 42
clopidogrel 81 shock 94, 96, 99
Clostridium spp. infections 141, 189 consultants
clotting factors, dilution 100 communication with 220
co-morbidity 19, 139 involvement 21–2, 103
diabetes mellitus 230–5 continuity of care 221
and surgical risk 228 continuous positive airway pressure (CPAP) 44
coagulopathy 100, 149, 173 continuum of care 5–6
Cockcroft Gault formula 135 COPD, see chronic obstructive pulmonary disease
codeine 189, 206 ‘Cori cycle’ 192
collegiality 219 corrected flow time (FTc) 117
colloid fluids 16, 100 corticosteroids 235
colonic anastomoses, leakage 141, 150–1, 172 cortisol 93, 229
colostomy, stoma management 153–4 cough, ability 42, 201
communication 214–15 CPAP, see continuous positive airway pressure
aggressive vs assertive 226 (CPAP)
bad news 216–18 creatine kinase 129
barriers to 216 creatinine
blocking 226 serum 123, 125
body language 218 urine/plasma ratio 130
checking back 225 cricothryoidotomy 14
colleagues/surgical team 214–15, 220–1 critical care setting 216
liaison person 218 cryoprecipitate 100
medical errors 218–19 crystalloid fluids 100
with patients 214, 216–19, 225–6 CT pulmonary angiography (CTPA) 49
problems arising from poor 214 CURB 65 score 49
questions 216, 225 CVD, see cardiovascular disease
compartment syndrome cyclizine 80
abdominal 142–3, 145–6 cyclo-oxygenase 2 (COX-2) inhibitors 205
decompression 146, 148 cytokines 168, 194, 229, 230
lower limb 147
complaints 214, 219 daily management plans 20–1
complementary approaches 2 DC cardioversion 73
complications, see postoperative complications decision making, daily plans 20–2

244
INDEX

decision-making 8, 219, 220 EA, see epidural analgesia


definitive care 24 ‘ebb and flow’ responses 229
delusions 223 ECF, see extracellular fluid
depression, patient 223 echocardiography, trans-thoracic/trans-oesophageal
deterioration, recognition 138, 140–1 117
dexamethasone 207 elderly patients
dextrose infusion 234 atrial fibrillation 72
hyperkalaemia 132 blood pressure 62
diabetes mellitus 230–5 risks of surgery 236–7
DKA 234–5 elective surgery 157
HONK 235 electrocardiograph (ECG) 66
hypoglycaemia 235–6 atrial tachycardias 75–6
type 1 231 hyperkalaemia 132
type 2 231, 234 interpretation 66–70
diabetic ketoacidosis (DKA) 233, 234–5 left bundle branch block 78
dialysis 130–1 left ventricular hypertrophy 77
diarrhoea myocardial infarction 79–81
enteral feeding 189 normal 70–1
fluid and electrolytes losses 92, 164 pulmonary embolism 49
diclofenac 208 respiratory failure 36–7
digoxin 76 right bundle branch block 78–9
disorientation, patient 216 right ventricular hypertrophy 77–8
diuretics 86, 133 routine for examining 66
loop 56, 86, 133 shock 98
thiazide 235 supraventricular tachycardia 74
DKA, see diabetic ketoacidosis ventriclar tachyarrhythmias 74–6
dobutamine 118, 119 electrolytes 65, 159–62, 195
dopamine 118 depletion 164
dopexamine 118 see also specific electrolytes
Doppler, trans-oesophageal (TOD) 116–17 emergencies, management 7, 8, 9, 156, 221
dorsalis pedis artery, cannulation 109 ‘emotional numbing’ 221–2
drains, management 154 emotions, patients 217–18
see also chest drain empathic statements 217–18, 225–6
drug charts emphysema, chest X-ray 41
cardiovascular assessment 64 endocrine factors, shock 93
pain management 202 endotoxins 185
dyspnoea 61 endotracheal tube 14, 27

245
CARE OF THE CRITICALLY ILL SURGICAL PATIENT

energy fluid balance


assessment of expenditure 194 chronic renal failure 135
requirements 185, 192–3 clinical assessment 157
enteral feeding renal failure 134
benefits 184–5 and renal perfusion 123, 124
complications 189 fluid balance charts 94, 157, 164, 165
contra-indications 186 fluid challenge 15, 98, 101, 103, 111, 210
routes 186–8 fluid compartments 157–8
enterostomy, tube 187 fluid management 15–16, 162–4
epidural analgesia (EA) 200, 204, 209–11 biochemical assessment 158–62
breakthough pain 211 case scenario 165–6
catheter insertion 209 central vein 114
drugs 209, 211 critical illness and emergency surgery 156
hypotension/vasodilation 93, 201, 202, 210, 211 elective surgery 157
troubleshooting 211 epidural analgesia 201
evidence-based medicine 237 renal failure 129
exposure, patient 17 sepsis 174–5, 177
extracellular fluid (ECF) 157–8 shock 97, 99–100
losses 163 fluid overload 157
replacement 164 cardiac failure 83–4
and sodium balance 159 renal failure 128
extracorporeal life support (ECLS) 47 fluids
colloid 16, 100
failure to progress 7, 8 crystalloid 100
fast-track recovery programmes 157 fresh frozen plasma 100
fasting frusemide 86, 133
chronic renal failure 135 full blood count
metabolic response 192–3 ARF 128, 129
fatigue, operational 216 respiratory failure 36
fats, dietary 185 functional residual capacity (FRC) 34
fatty acids 192, 234 fungal infections 173, 180
fear 222
femoral artery cannulation 109 gallop rhythm 95
femoral vein, intravenous feeding 190 gastric emptying 186–7
fentanyl 206, 209, 210 gastric residual volume 187, 188
fibrinolytics 81 gastrointestinal fluid, loss 92, 164
fistulae, intestinal 151–2, 164, 190

246
INDEX

gastrostomy Harris–Benedict equation 194


percutaneous endoscopic (PEG) 188 Hartmann’s solution 164
stoma management 153 Health Act 2006 Code of Practice 115
tube 187 heart failure, see congestive cardiac failure
glicazide 233 heart rate 62
glomerular filtration rate (GFR) 122, 123 haemorrhagic shock 95
acute renal failure 125 heart rhythm 62
chronic renal failure 135 heart sounds 61, 64, 95
glucagon 193, 229 heparin 49, 81, 209
gluconeogenesis 229 hepatic ischaemia 145
glucose high dependency unit (HDU) 2, 5–6, 18–19
blood levels 229, 235–6 high-impact intervention approach 115
as energy source 185 Hinton, John 217
glucose–potassium–insulin (GKI) infusion 231, 232 history 19
glutamine 192, 193, 229 cardiovascular disease 64
glutathione 229 renal failure 128
glycerol 192 respiratory failure 36
glyceryl trinitrate 80, 133 sepsis 171
glycogen stores 192, 229 HONK, see hyperosmolar non-ketotic state
glycogenolysis 192, 193 hormones
glyconeogenesis 193 counter-regulatory 193, 229, 230, 235
GMC Good Doctor Guidelines 219 shock 93
Guedel airway 14, 27, 28 hospital-acquired infections 48, 154, 175–6, 179
hydrogen ion concentration 55, 160, 164
haematuria with casts 128 hypercalcaemia 161
haemodialysis 131 hypercapnia 34, 36, 53
haemofiltration 131 permissive 46
haemoglobin (Hb) 65, 130 hyperchloraemic acidosis 191
haemorrhage 15–16, 92 hyperglycaemia 191, 232, 233
classification of blood loss 92 hyperkalaemia 131, 132, 160
coagulopathy 149 ECG changes 132
postoperative 16, 19, 148–50 emergency therapy 132
reactive 149 hypermagnesaemia 161
secondary 150 hypermetabolism 194
tracheostomy site 31 hypernatraemia 159
haemorrhagic shock 15–16, 91, 92, 95, 101 hyperosmolar non-ketotic (HONK) state 235
hallucinations 223 hypertension 87, 201
hand washing 154 intra-abdominal 145
handover 221 hypertriglyceridaemia 191

247
CARE OF THE CRITICALLY ILL SURGICAL PATIENT

hypocalcaemia 161 inotropic agents 118–19


hypoglycaemia 17, 191, 235–6 actions 118
clinical events 236 safety 119
factors contributing 235 shock 103, 104
hypokalaemia 56, 70, 160, 166 inspiratory to expiratory (I:E) ratio 46
hypomagnesaemia 161 insulin 192, 193
hyponatraemia 158–9 insulin infusion 132, 234
hypophosphataemia 162 sliding scale 232
hypotension 62, 71–2, 86 insulin resistance 193, 229
epidural analgesia 201, 202, 211 intensive care unit (ICU) 2, 3, 5–6
sepsis 177 anticipating need 140
hypothermia 99, 100 assessing abdomen 140–1
hypotonic solutions 234 discharge 47–8
hypovolaemia 7, 15, 60, 61, 64 recognising deterioration 140
renal failure 128 shocked patient 103, 104
hypovolaemic shock 91, 92, 95, 101 interleukin-1 194, 229, 168
hypoxaemia 53, 57, 61, 201 interleukin-6 168, 194, 229, 230
hypoxia 26, 107 internal jugular vein, cannulation 113, 190
definition 34 intestinal failure 195–6
intestinal fistulae 151–2, 164, 190
iatrogenic factors intestinal hypoxia 90, 107
hypovolaemic shock 92 intestinal ischaemia 139, 145
see also medical mistakes intestinal obstruction 190
ileostomy, management 153–4 intestines, benefits of enteral nutrition 184–5
immunocompromise 173–4 intra-abdominal hypertension (IAH) 145
incisions, pain associated 199–200 intravascular volume 115, 128
infections intravenous lines, management 63
catheter-related 115, 176, 180, 191 intubation 14, 27, 45
Clostridium spp. 141, 189 investigations
fungal 173, 180 post-operative complications 141
hospital-acquired 48, 154, 176, 179 post-operative pain management 203
hypovolaemia 92 review 20
MRSA 176 targeted 21–2
respiratory 37 ion exchange resin 133
surgical site 154, 238 iprotropium 37
information, key 220 ischaemic heart disease 79–82, 85
inhalers 37 risks of surgery 86–7
injury, metabolic response 193, 229–30 isoprenaline infusion 79

248
INDEX

jaundice, obstructive 127 local anaesthetics


jejunostomy, tube 187 epidural 209, 210
Joint College Working Party Report on intra-operative infiltration 200, 204, 211
Postoperative Pain 200 loperamide 189
jugular venous pressure (JVP) 62, 84, 95, 96 low molecular weight heparin 49, 81, 209
lung disease, chronic 41, 43, 44, 53
Kerley B lines 41, 42, 84 lung function tests 37
ketone bodies 192 ‘lung protective ventilatory strategy’ 46
ketones 234
kidneys magnesium 65, 161
functions 122 malnutrition 184, 192
imaging 128, 129 manometer, liquid 113, 114
perfusion 63, 122, 123, 129 mean arterial pressure (MAP) 122
potassium excretion 160 medical mistakes 218–19
see also renal failure meningococcal sepsis 93
metabolic acidosis 55, 56, 57, 58, 104
lactate, serum 54, 96, 103, 107, 177 causes 56
language 217 DKA 234–5
laparoscopic surgery 200 metabolic alkalosis 56, 58
laparotomy 141, 146, 176 metabolic chart, bedside 194
laryngeal mask airway 27 metabolism
leadership 220–1, 239 refractory shock 103–4
left bundle branch block 78–9 response to sepsis 194
left ventricular hypertrophy 77 response to surgery/injury 193, 229–30
left ventricular stroke work index (LVSWI) 116 and starvation 192–3
limb compartment syndrome 147, 148 metformin 233, 235
line sepsis 191 metoclopramide 80, 207
lipolysis 192, 229 MEWS charts 38
listening skills 217 microbiologist 179
lithium chloride dilution calibration system minerals 185
(LiDCO ™) 117 minute volume 45
litigation 214, 219 ‘mirroring’ 218, 226
liver mitral valve disease 70
dysfunction in TPN 191 mobilisation 198
metabolism 192, 229, 230 MOF, see multiple organ failure
palpation 65 monoclonal antibodies 179
liver function tests (LFT) 129, 173 morphine 80, 206

249
CARE OF THE CRITICALLY ILL SURGICAL PATIENT

mortality nursing staff


national surveys 237 communication with 220–1
renal failure 122 HDU/ICU 5
septic shock 168 pain management 202–3, 212
MRSA infections 176 tracheostomy management 130
multidisciplinary teams, pain management nutritional prescription 185
200, 208 nutritional status, assessment 194–5
multiple organ failure (MOF) 2, 107, 168, 177, nutritional support
179, 181 advanced theory and practice 192–4
muscle, skeletal 192, 229 calculating requirements 185
myocardial infarction (MI) 79–81 duration 185
acute management 81 enteral
anterior 82 benefits 184–5
inferior 81 complications 189
risk of peri-operative 87 contra-indications 186
myocardial ischaemia 201 initiation 188–9
myoglobinuria 130 routes 186–8
indications 184–5
naloxone 207 sepsis 175
National Confidential Enquiry into Patient total parenteral 190–1
Outcome and Death (NCEPOD) 237
National Institute for Health and Clinical obesity 236
Excellence (NICE), central venous access 113 obstructive shock 91, 92–3, 96, 97
National Patient Safety Agency 187 oliguria 96, 123, 124, 145
nausea 207 omeprazole 189
nebulisers 37 ondansetron 207
necrotising fasciitis 150 operation notes 138–9, 144
negligence 214 operational fatigue 216
nephrology referral 133 opioids 206
nephrotoxins 126–7, 130, 179 antagonist (naloxone) 207
nerve blocks 204, 211 intramuscular 206
neutropenia 141 intravenous 206
nitrates 80, 119 myocardial infarction 80
nociception, defined 198 oral 206
non-steroidal anti-inflammatory drugs (NSAIDs) side effects 201, 206–7
204, 205 slow-release 206
noradrenaline 118, 119 toxicity 62, 202, 205

250
INDEX

organic reactions, acute 223 local anaesthetic nerve/plexus blocks


organisational skills 47–8, 219–20, 221 200, 204, 211
outcomes multidisciplinary team 200, 212
adverse 3, 214 multimodal 205
audit 237 patient assessment 200–3
patient expectations 214 patient controlled (PCA) 207, 209
scoring systems 228, 237–8 pre-emptive 199
oxygen principles 199–200
arterial partial pressure (PaO2 ) 34, 35, 43, 52 reasons for 198
inspired concentration (FiO2 ) 14, 46–7, 53 and respiration 42, 201, 208
saturation of haemoglobin (SaO2 ) 17, 35–6, techniques available 203–5
52, 133, 208 ward/nursing provision 212
oxygen delivery (DO2) 106–7, 116 pain scoring systems 202–3
oxygen dissociation curve 34, 35, 52, 53 pancreatitis 187
oxygen therapy paracetamol 204, 205
analgesia 201, 208 passivity 226
failure 43 patient assessment
masks 26–7, 42 acutely unwell patient 13–17
sepsis 175 cardiovascular system 64–5, 106
shock 97 case scenarios 19, 22
see also ventilation CCrISP system 12, 24
oxygen uptake (VO2) 116 chart review 18–19
circulation 15–16, 61
pacemakers 87 end of immediate management 17
pain fluid balance 157
‘breakthrough’ 199, 211 full 18–20
chronic 211 immediate (ABCDE) 13–17
definition 198 nutritional status 194–5
role of 198 pain management 200–3
pain management re-assessment 4, 16, 23, 220
aims of 199 renal failure risk 126–7
analgesic agents 205–7 ‘stable’ surgical patient 12, 18
analgesic ladder 204, 205 surgical site 138–44
case scenarios 199, 208, 210 unstable patient 21–2
epidural 200, 204, 209–11 patient controlled analgesia (PCA) 207, 209
escalating requirements 199 patient expectations
pain relief 199
surgical outcomes 214
‘Patient Safety First’ 238

251
CARE OF THE CRITICALLY ILL SURGICAL PATIENT

percutaneous endoscopic gastrostomy (PEG) replacement in diabetes 234


153, 188 requirement 160
perfusion 62, 90, 106–7 potassium chloride 234
peripheral 15, 64, 96 practical skills
renal 122, 123, 124, 129 chest drain insertion 50
pericardial effusion 41–2 chest X-ray interpretation 40–2
peripherally inserted central (PICC) line 190 pre-operative assessment
peritoneal lavage 176 nutritional status 194–5
pH (arterial blood) 54, 160 renal function 126–7
phenformin 235 pressure support ventilation (PSV) 45
phosphate 161–2 preventative actions 3
physiotherapy, respiratory 4, 37, 38, 43, 48, 208 case scenarios 4, 7
‘pink sign’ 148 renal failure 126–7
platelet transfusion 100 respiratory failure 39
pleural effusion 41 sepsis 176–7
plexus blocks 204, 211 prochlorperazine 207
pneumonia 48–9 prognosis, communication with patient 216–18
pneumothorax protein catabolism 194, 229, 230
chest drain 50 protein metabolism 229, 230
tension 46, 50, 63, 96, 115 protein-energy malnutrition (PEM) 184, 194
positive end expiratory pressure (PEEP) 45–6 proteinuria 128
post traumatic stress disorder (PTSD) 222 pseudo-aneurysm formation 52
postoperative complications 138 psychiatric referrals 223–4
abdominal compartment syndrome 145–6 psychological disorders 222–3
anastomotic leakage 141, 150–1, 172 psychosis 223
burst abdomen 148 pulmonary angiography, CT (CTPA) 49
case scenarios 142, 147, 149 pulmonary artery catheter
examples and initial presentations 139 (PAC/Swan–Ganz catheter) 116
haemorrhage 16, 19, 148–50 pulmonary embolism 49–50, 96
lower limb comparment syndrome 147, 148 pulmonary oedema
necrotising fasciitis 150 cardiogenic 84
warning signs 141 iatrogenic 64
potassium 65, 195 renal failure 128, 133
depletion 164 signs and management 133
low plasma (hypokalaemia) 56, 70, 160, 166 pulmonary secretions, management 3, 4, 37, 38,
raised plasma (hyperkalaemia) 131, 132, 160 43, 48, 208
reducing total body 133 pulmonary sepsis 175
renal excretion 160 pulmonary vascular dysfunction 34
pulse contour cardiac output with indicator

252
INDEX

dilution (PiCCO) 117 renal failure index 130


pulse oximetry 14, 17, 35–6, 98 renal function 122–3
limitations 36, 201, 203 and acid–base balance 55
mechanism 35 ‘biomarkers’ 135
pulse pressure 95 renal perfusion 122, 123, 124, 129
pulse rate 62 renal perfusion pressure 122, 123
pulses 15, 61, 62 renal plasma flow (RPF) 122, 123
renal replacement therapy (RRT) 130–1
quality of care 228 renal transplant 135
questions 216, 225 renin–aldosterone system 160
respiration
radial artery cannulation 52, 109, 110 acid–base balance 55, 57
radiography assessment 14–15
renal failure 128, 133 common surgical problems 48–50
see also chest X-ray and CVS 61
randomised controlled trials (RCTs) 237 depression in opioid use 206–7
RCS’S mnemonic 40 and pain management 42, 201, 208
re-assessment 4, 16, 23, 220 respiratory acidosis 55, 56, 104
re-operative surgery 140, 141, 151 causes 56
re-perfusion injury 148 respiratory alkalosis 55, 56
red cell concentrates 99 respiratory failure 15
refeeding syndrome 185, 191 assessment 35–7
relatives 214, 216 case scenarios 38, 43
renal failure causes 34
aetiologies 125–6 clinical signs 35
case scenarios 124, 127, 134 definition 34
chronic 129, 135 discharge from ICU 47–8
common scenarios in surgical patients 126 management 42–5
complications 132–3 in MOF 177
golden rules 123 prevention 39
intrinsic 129, 130 stable patient plan 37–8
investigations 128–9 ventilation 45–7
management 126–31 respiratory rate 62
in MOF 179 restlessness 96
nephrology referral 133 results, review 20, 65
post-renal 126, 129 rhabdomyolysis 130
pre-renal 129, 130 right bundle branch block 78–9
prevention 126–7 right ventricular failure 114
in severe sepsis 173 right ventricular hypertrophy 77–8

253
CARE OF THE CRITICALLY ILL SURGICAL PATIENT

risk assessment 237–8 definition 90


see also surgical risk fluid administration 15–16, 99–100
roziglitazone 235 haemorrhagic/hypovolaemic 15–16, 91, 92,
95, 101
salbutamol 37, 38, 132, 160 ICU transfer 103, 104
saline, nebulised 42 inotropic support 103, 104
sedation, opioids 201, 207 management principles 97
Seldinger technique 110, 112 monitoring and instrumentation 97–8
senior colleagues neurogenic 93
communication with 220 obstructive 91, 92–3, 96, 97
involvement 21–2, 103 patient categories 16
sepsis refractory 102, 103–4
abdominal 171, 175, 176, 178 responses to treatment 100, 102
catheter-related 180, 191 septic 90, 91, 93, 94, 96–7, 100, 169, 177
causes 170–1 short bowel syndrome 189, 190
definitions 169–70 simulation, critical incidents 238
fungal 173, 180 ‘sip feeds’ 186
management 174–6 SIRS, see systemic inflammatory response
mediators 168, 179 syndrome
metabolic response to 194 skeletal muscle 192, 229
patient assessment 171–5 skin
patients at risk 168 examination 65
prevent, diagnose, act 176–7 perfusion 94, 96
severe 169, 173 smoking 48
Sepsis Six 176 SNAP protocol 151–2
sepsis syndrome 169, 170, 174 sodium 159–60, 195
septic shock 90, 91, 93, 94, 169, 177 basal requirements 159
bacteriological samples 99 fractional excretion 130
clinical features 96–7 low serum (hyponatraemia) 159–60
management 100 raised serum (hypernatraemia) 159
mortality 168 renal excretion 129, 157–8
shock 15–16 urinary levels 130, 159
aetiology 90–1 sodium bicarbonate infusion 130, 132
blood pressure 90, 95, 97 sodium nitroprusside 119
cardiogenic 86, 92, 95, 97 sphygmomanometer, automated 107
cardiovascular monitoring/support 102–3 sputum culture 203
case scenarios 93, 101 sputum samples 37
clinical features 15, 94–7 stable patient 12, 18
continuing assessment 99 staff relationships 219, 220

254
INDEX

Stamm technique 187 systemic inflammatory response syndrome (SIRS)


Starling curve 84, 92, 118, 119 169–70
starvation, metabolic response 192–3 causes 170–1
stoma management 153–4 definition 169
streptococci, enteric 175 systemic vascular resistance (SVR) 93, 96–7, 116
streptokinase 81
stress, in critical care 2, 221–2 tachycardia 62, 96, 201
stress response 193 causes 72
stroke index (SI) 116 post-surgical patient 201, 202
stroke volume (SV) 92, 95, 111, 116 sinus 75
sub-optimal care 3 tachypnoea 44, 57, 96
subclavian vein, infraclavicular, cannulation team skills 214–15
112, 190 temperature 63
suffering 198 core–peripheral gradient 63, 94, 99
suicidal thinking 223 measurement in shocked patient 99
sulphonylureas 230 postoperative pyrexia 172
‘super-infections’ 175 sepsis/SIRS 63
supraventricular tachycardia (SVT), paroxysmal 75 thiazolidinediones 235
surgery, metabolic response 193, 229 thinking on the run 9
surgical handover 221 thyroxine (T4) 193
surgical risk tidal volume (Vt) 45, 46
assessment 237–8 tissue perfusion 106–7
cardiovascular disease 86–7 shock 97
diabetes 230 signs of poor 94
elderly patients 236–7 tonometry 107
factors increasing 2, 6–8, 228 total parenteral nutrition (TPN) 190–1
and obesity 236 trace metals 162
reduction initiatives 238 tracheostomy 28
surgical site, postoperative assessment 138–44 complications 30–1
surgical team, communication 214–15, 220–1 indications 28
surgical trainee, role and responsibilities 8, 219–20 tube removal 31–2
Surviving Sepsis Campaign (SSC) 176–7 types of tube 28–9
suxamethonium 160 tramadol 206
Swan–Ganz catheter 116 trans-oesophageal Doppler (TOD) 116–17
sympathetic blockade 93, 201, 211 transducers 107–8, 111, 113
sympathetic statements 218 transfusion 100
synchronised intermittent mandatory ventilation trauma
(SIMV) 45 penetrating abdominal 139
syndrome of inappropriate ADH secretion 159 shock 92

255
CARE OF THE CRITICALLY ILL SURGICAL PATIENT

traumatic events 221–2 pressure controlled inverse ratio (PCIRV) 46–7


triglycerides 192 synchronised intermittent mandatory (SIMV) 45
triiodothyronine (T3) 193 weaning 47
troponin levels 65 ventilator-associated pneumonia 48
tumour necrosis factor 168, 229 ventricular ectopics 74–5
ventricular filling pressure 92
ultrasound ventricular tachycardias 74
abdominal 141 villous atrophy 185, 189
central venous catheter placement 112, 113 visceral perfusion 90, 107
renal 128, 129 vital signs, sepsis 171
unstable patient, management 21, 22 vitamin D 161
uraemia 128, 131 volutrauma 46
urea, urine/plasma ratio 130 vomiting
urinalysis 128 fluid and electrolyte losses 92, 164
urinary anastomoses 165–6 opioid analgesia 207
urinary catheter
infection 176, 180 ward
shock management 98 pain management 212
urinary osmolality 130 tracheostomy management 30
urinary retention 7 ward rounds 8, 18, 20, 219, 220
urinary tract obstruction 126, 128 water
urine output 63, 94, 96 abnormal losses 160, 162
acute renal failure 125 basal requirements 158, 162
normal adult 123 oral intake 162
urine specific gravity 130 total body volume 157, 158
urine/plasma osmolality ratio 130 ‘water’ manometer 113, 114
Waterhouse–Friedrichsen syndrome 93
Valsalva manoeuvre 72, 74 ‘wedge’ pressure 116
vasoconstriction, peripheral 95 weekend on call, communication and organisation
vasodilation 215
epidural analgesia 210, 211 weekend treatment 3, 4
shock 91 weight loss, pre-operative 236
vasodilators 114, 119 wheeze 37
vasopressors 119 white cell count 141, 172, 173
ventilation 45–7 Witzel technique 187
CPAP 44 World Health Organization (WHO)
lung injury 46 analgesic ladder 204, 205
non-invasive by mask (BIPAP) 45 Surgical Safety Checklist 238
wound management 154

256

You might also like